You are on page 1of 128

1

Light reflected by interface

A mixture of red light (vacuum = 661 nm) and blue light (vacuum = 472 nm)
shines perpendicularly on a thin layer of gasoline (ngas = 1.40 lying on water
(nwater = 1.33). The gasoline layer has a uniform thickness of 1.69 107 m.
What color is the light reflected from the gasoline? Neglect any effects(s) caused
by the material under the water.
Solution
Because the light wave reflected from the air/gas interface is inverted, and that from
the gas/water interface is not (all transmitted waves are transmitted in phase), we will
get destructive interference when the path difference of the two waves in the gasoline
is an integral number of wavelengths, and constructive interference when it is a halfintegral number of wavelengths. The path difference is always twice the thickness of
the gasoline layer for normal incidence. The wavelength of light in a medium is
=

c/n
vacuum
=
,

since the time frequency of a transmitted wave cannot change (think of a particle on
the boundary responding to the incoming EM wave) and the speed of light in the
medium is reduced by dividing by n. This means that the number of wavelengths of
light in the path difference = 2t is
N=

2tn
,
vacuum

which is 0.716 for the red light and 1.00 for the blue light. This means that the
interference for the blue light is destructive and we will see predominantly red light.
Note that since the intensities of the two reflected blue waves will not exactly match
because of the different ratios of the indices of refraction at the two boundaries, we
will not get a perfect cancellation of the reflected blue light.

Problem OpaTh-F99-12 from Courses-SC.

Specific heat of magnetic solid

A solid contains N magnetic atoms having spin 12 . At sufficiently high temperature, each spin is completely randomly oriented, i.e. equally likely to be
in either of its two possible states. But at sufficiently low temperature, the interaction between the magnetic atoms causes them to exhibit ferromagnetism,
with the result that all their spins become oriented along the same direction as
T 0. A very crude approximation suggests that the spin-dependent contribution C(T ) to the heat capacity of this solid has an approximate temperature
dependence given by

(


1
if T1 /2 < T < T1
C1 2T
T1
C(T ) =
0
otherwise
Use entropy considerations to find the maximum possible value of C1 .
Solution
If we know the specific heat we can find the entropy by using the specific heat to
find the heat added, and then using that the entropy is related to the heat added by
dQ = T dS. For a system of spins we can also find the entropy using the number of
microstates available to the system. If we relate the entropies found in these two ways
we can find a relation for C1 .
If dQ = T dS and dQ = C(T )dT , then we have that the relationship between entropy
and specific heat is that
C(T )
dT,
dS =
T
so that we can integrate to find the entropy S
S(T 0 ) =

T0

Z
0

C(T )
dT.
T

If we choose a final temperature T large compared to T1 , then we have


S(T 0 )

Z
=
=
=
=

T0

C(T )
dT
T
0


Z T1
1
2
C1

dT
T1
T
T1 /2

T1
2T
C1
ln(T )
T1
T1 /2
C1 [1 ln(2)] .

Now when the system is at very low temperature the spins are all aligned and the
number of microstates available to the system is very smalljust 2! This means the
entropy is simply
S = kB ln() = kB ln(2)
Problem OpaTh-F00-2 from Courses-SC..

at very low temperature. As we raise the temperature a few of the spins will be able
to be misaligned, until at very high temperature all spin configurations (including half
of the spins aligned one way and half the other way, for example, and the states where
all the spins are aligned) are possible. If the spins are aranged in a regular lattice and
can be labeled, then the largest possible number of available microstates of the system
is given by two choices for spin 1, times two for spin 2, etc, for a total of = 2N , so
that the maximum value of the entropy is
S = kB ln() kB ln(2N ) = N kB ln(2).
Comparing this to our result above, which is also for high temperature, we have
C1 [1 ln(2)] N kB ln(2),
so that
C1 N kB

ln(2)
.
1 ln(2)

Two converging lenses

Two converging thin lenses, L1 and L2 , each of focal length f1 = f2 = 10 cm,


are separated by the distance d = 35 cm. An object (the upright arrow) is
placed a distance O1 = 20 cm to the left of the left-hand lens (L1 ). See the
figure. The check marks on the horizontal axis represent the focal points of the
two lenses.

(a) Draw a ray diagram, and also find the position of the final image using the
thin-lens equation.
(b) Is the final image real or virtual?
(c) Is the final image upright or inverted?
(d) What is the magnification of the final image?
Solution
(a) Since the two focal lengths are both the same, call them both f . Then we can
use the thin lens equation 1/s + 1/s0 = 1/f to find the exact positions of the
two images, which are shown in the following ray diagram. Note we have used
the two rays from the tip of the object through the focal point (which comes
out parallel) and through the center (which is undeflected) of the lens to locate
the tip of the image, and the base of the image is located by a ray along the
axis of the two lenses.
From the lens equation, using the distances labeled on the figure, we have
1/s01 = 1/f 1/s1 = (1/10 1/20) cm1 ,
so that s01 = 20 cm. This means that s2 = 35 20 = 15 cm, and so


32
1/s02 = (1/10 1/15) cm1 =
cm1 = 1/30 cm1 ,
30
so that s02 = 30 cm.
Problem OpaTh-F00-2 from Courses-SC.

(b) Obviously the final image is real, as rays of light converge to a point at the image
where we can hold a screen.
(c) The final image is obviously upright.
(d) For this we need to calculate the size of the intermediate image from the magnification m1 = s01 /s1 = 20/20 = 1, so we see this image is of the same size as
the object but inverted. The second image has magnification m2 = s02 /s2 =
30/15 = 2, so the combined magnification is m = m1 m2 = 2.

Ideal gas in confining potential

An ideal classical gas made of N point-like atoms with mass m is placed in


a confining potential: V (x, y, z) = 12 (kx x2 + ky y 2 + kz z 2 ). The temperature of
the gas is T and there is no external container.
(a) Calculate the average kinetic energy of each gas atom.
(b) Calculate the average potential energy of each gas atom.
(c) Calculate the constant volume heat capacity of the gas.
(d) At low temperatures, the gas has to be considered as composed of quantum
mechanical particles, and the fact that their energy levels are discrete
is important. Briefly discuss what happens to the specific heat at low
temperatures (no calculation is required).
Solution
(a) The equipartition theorem states that for every independent term quadratic in
the dynamical variables (which are momenta and positions) in the Hamiltonian
(energy) of the system we have an average energy of kT /2. The kinetic energy
of each gas atom has three such terms, which are p2x /2m, p2y /2m, and p2z /2m.
Each term has associated with it an average energy of kT /2, so the average
kinetic energy of each gas atom is
h

p2i
3
i = kT.
2m
2

(b) Exactly the same argument holds for the potential energy of each particle, which
contains terms quadratic in the independent position variables x, y, and z. so
that for any i
3
hV (ri )i = kT.
2
(c) For an ideal gas heated at constant volume, which can do no work, the first law of
thermodynamics tells us that the heat capacity CV is derivative of the internal
energy U of the gas with temperature T . The internal energy is the sum of
the average kinetic and potential energies of each gas particle multiplied by the
number of such particles,


3
3
U =N
kT + kT = 3N kT,
2
2
and so
CV =

dU
= 3N k.
dT

(d) The third law of thermodynamics states that the entropy S goes to a constant
S0 as the temperature goes to zero,
lim S = S0 .

T 0

Problem OpaTh-S00 from Courses-SC.

We can understand this microscopically by the number of available microstates


of the system tending to a constant at absolute zero, and the entropy is related
to the natural log of the number of available microstates. Now since the heat
capacity is defined to be


dQ
CV =
dT V
and since dQ = T dS, we have

CV = T

dS
dT


.
V

But S approaches a constant as T 0, so we have


lim CV = 0.

T 0

Although somewhat beyond the scope of this question, we can establish how
the specific heat tends to zero. Examine the energy of a single particle in a
one-dimensional harmonic potential. The partition function is
X E

,
Z=
e

where = 1/kT and the energy eigenvalues E are simply (n + 1/2)


h, where
is the harmonic oscillator frequency. For this case the partition function is
just a geometric series with the sum
Z=

e(n+1/2)h = eh/2

enh =

n=0

n=0

eh/2
.
1 eh

Now the average energy of the system is


hEi =

Now we have that



1 X
1
Z
ln Z

=
.
E eE =
Z
Z

1
ln(Z) =
h ln(1 eh ),
2

so that
U = hEi =

ln Z
1
heh

=
h +
=
h

2
1 eh

1
1
+ h
2
e
1


.

The first temperature-independent term is the zero-point energy of the oscillator.


The generalization to N particles in three dimensions is simpleeach dimension
in which each particle moves behaves like an independent one-dimensional oscillator, so that the energy is just multiplied by 3N ,

U = hEi3D = 3N
h

1
1
+ h
2
e
1


,

Here we assume kx = ky = kz for simplicity, and now by a straight-forward


differentiation we have that
 h

h

e
h

CV = 3N
,
kT 2 (eh 1)2
and as the temperature goes to zero we see that

2
h

lim CV = 3N k
e kT .
T 0
kT
We see that, since the exponential goes to zero faster than any power of T , the
specific heat goes to zero exponentially. This is a characteristic of any system
with an energy gap such as this one (which has an energy gap of h
between
the ground state and first excited state). Note the high temperature limit of Cv
produces the ideal gas Cv = 3N k.

Expansion of ideal gas

Consider a reversible isothermal expansion of an ideal gas (the system) with


N particles in contact with a heat reservoir at a temperature T , from an initial
volume V1 to a final volume V2 .
(a) What is the change of the internal energy of the system?
(b) Obtain the amount of work performed by the system.
(c) What is the amount of heat absorbed by the system?
(d) What is the change of entropy of the system?
(e) What is the change of the entropy of the system plus the reservoir?
Next consider a free expansion of the above ideal gas from an initial volume
V1 to a final volume V2 .
(f) What is the change of temperature of the system?
(g) Does the internal energy change?
(h) Obtain the amount of work performed by the system.
(i) Obtain the absorbed amount of heat.
(j) Obtain the change of entropy of the system and that of the reservoir.
Solution
(a) The internal energy of an ideal gas U = CV T = N cV T , where cV is the specific
heat per particle and CV is the total specific heat, depends only on the number of
gas molecules and the temperature of the system, so in an isothermal expansion
where the temperature (and amount of gas) remains the same, the internal
energy does not change, i.e. U = 0.
(b) The work done is
Z

V2

W =

P dV,
V1

and since for an ideal gas at a fixed temperature T we have P = N kB T /V we


can write
 
Z V2
V2
dV
= N kB T ln
.
W = N kB T
V
V1
V1
(c) Using the first law of thermodynamics, Q = U + W , and since U = 0 we have
 
V2
Q = W = N kB T ln
.
V1
Problem OpaTh-FS01 from Courses-SC.

(d) The change in entropy is


Z
Z
Z
dQres
1
Qres
S = dS =
=
dQres =
,
T
T
T
since the temperature T does not change, and so
 
V2
S = N kB ln
.
V1
(e) Since the temperature of the heat reservoir does not change by assumption, then
since it loses heat Q to the gas we know that its entropy change is just
 
V2
Qres
Q
Sres =
=
= N kB ln
.
T
T
V1
Note that this means that the total entropy change is zero, which is the definition
of a reversible process.
(f) In a free expansion, such as a vessel of volume V1 separated from one of volume V2
by a membrane which ruptures, the ideal (and so non-interacting) gas molecules
have the same kinetic energy before and after the membrane ruptures, so that
the temperature remains the same.
(g) No, since it depends only on the temperature and that has not changed. Alternatively, if you think of the motion of each of the molecules before and after the
membrane ruptures, the kinetic energy of the molecules has not changed and so
neither has U .
(h) When a membrane ruptures, even though V has changed, there is no work done
by the system on its surroundings.
(i) By the first law of thermodynamics, if there is no work done and no change in the
internal energy there can be no heat absorbed.
(j) If there is no heat absorbed from it, then there can be no change in the entropy of
the reservoir, Sres = 0. The gas, however, has now more available microstates
(it has the same total energy but the energy levels are now closer together) and
so has more entropy. Another way of saying this is that the entropy of the gas
is a function of the state of the gas, and if it has the same state as after the
isothermal expansion in Part 1 (d), then it has the same entropy, and so once
again
 
V2
S = N kB ln
.
V1
Then the total entropy of the system has increased, as expected for an irreversible process.

Diameters of wires

The diameters of fine wires can be accurately measured using interference


patterns. Two optically flat pieces of glass of length L are arranged with a wire
between them as shown in the figure. The setup is illuminated with normally
incident yellow light of wavelength = 590 nm. The length is L = 20 cm, and
19 bright fringes are seen along this length.
(a) Find the diameter of the wire.
(b) If the 19th fringe is not quite at the end, but there is no 20th fringe, give an
estimate of the possible relative error in the measurement of the diameter.
Solution
(a) Where the two plates touch, the light which reflects from the bottom of the top
piece of glass and the light which reflects from the top of the bottom piece of
glass travel have the same path length. However, the light from the second
reflection has its phase inverted by since it goes from a region of low density
to one of high density, while the opposite is true for the second reflection so it
is in phase. That means that these two reflected rays negatively interfere and
there is a dark fringe, as shown. As we move along the top plate, the path
length difference must increase by /2 (for a phase difference of ) by the time
we get to the first bright fringe, and then to (for a phase difference of 2)
by the time we get to the second dark fringe, and 3/2 for the second bright
fringe, and so on. By the time we get to the nineteenth bright fringe, we must
have a path length difference of (18 + 1/2) = 37/2 when we get to the 19th
bright fringe. Since the path difference is twice the separation of the plates
and so twice the diameter of the wire (with the excellent approximation that
the angular separation of the plates is very small so we have almost normal
incidence), then
D = 37/4 = 5.47 106 m.
(b) The dark fringe spans a path length difference of /2, so we can make a relative
error of a part in 37, or 2.7%.

Problem OpaTh-FS01-2 from Courses-SC.

Otto cycle 2

The operation of a gasoline engine is approximately represented by the Otto


cycle. This cycle consists of four different processes:
A B The gas is compressed adiabatically from volume VA to volume VB (compression stroke).
B C The gas is heated at constant volume VB (combustion of the gasoline
mixture).
C D The gas expands adiabatically from volume VB to volume VA (power
stroke).
D A The gas is cooled at constant volume VA (exhaust stroke).
(a) Compute the thermodynamic efficiency of the Otto cycle as a function of the
compression ratio VA /VB and the heat capacity per particle at constant
volume, cV . Assume that the gas is ideal with temperature-independent
heat capacities.
(b) Do higher or lower compression ratios give higher efficiency?
Solution
(a) During the constant-volume heating and cooling, by the first law of thermodyProblem OpaTh-FS01-3 from Courses-SC.

namics (and since no work can be done by or on the gas) we have that
Z T2
Z 2
dT = N CV (T2 T1 ),
dU = N CV
Q = U =
T1

so that we have
Qout = N CV (TD TA ), Qin = N CV (TC TB ),
where these are the (positive) heats that are given up and put into the system,
respectively. The thermodynamic efficiency of a cycle is defined to be the work
done by the gas divided by the heat put in,
=

W
,
|Qin |

and since the gas does not change its state over an entire cycle we know that its
internal energy does not change. The first law of thermodynamics then tells us
that the net heat added to the system is the work done by the system, so that
W = |Qin | |Qout | and the efficiency is


|Qin | |Qout |
|Qout |
W
TD TA
=
=
=1
= 1
.
|Qin |
|Qin |
|Qin |
TC TB
In an adiabatic expansion or compression we have the result (you should know
how to prove this!) that
T V 1 = constant,
where = CP /CV is the ratio of the specific heat at constant pressure to that
at constant volume. This will allow us to relate the (unknown) temperatures to
the (known) volumes and so compute the efficiency.
Note that
TC VC1 = TD VD1 ,
so

TC = TD

VD
VC

1


, TB = TA

VA
VB

1
,

From the diagram we see that VD /VC = VA /VB which is the compression ratio
r. Then we have that

 


TD TA
TD TA
= 1
= 1
= 1 r1 .
TC TB
(TD TA )r1
Now for an ideal gas we have that CP and CV differ by nR = N kB (this can be
derived from the first law of thermodynamics), so
1=

nR
N kB
kB
=
=
,
CV
N cV
cV

since R = NA kB and nNA = N , and so, finally,


 = 1 rkB /cV .
(b) Because 1 = kB /cV is a positive definite number for any gas,  = 1 r1 can
only get larger as r gets larger, meaning that higher compression ratios always
give higher efficiencies.

Light through film

A loop of wire is dipped into soapy water and forms a soap film when removed.
The loop is held so the film is vertical. The index of refraction of the film is 1.4
and the light incident on the film has a wavelength of 560 nm in vacuum.
(a) What is the wavelength of the light inside the film?
(b) If the thickness of the film is t = 106 m, find the number of wavelengths
contained in a thickness 2t.
(c) What happens to the phase of a light wave when it is reflected from
i) the front of the film, and
ii) the back of the film?
(d) As time progresses, the film becomes thinner due to evaporation. The film
then appears black in reflected light. Briefly explain why this occurs.
Solution
(a) Recall that the the speed of light in a medium with refractive index n is c/n. Then
= c/n and since cant change (the time taken for one oscillation cannot be
altered by a medium or waves would pile up) then = vacuum /n = 400 nm.
(b) 200 106 /(400 109 ) = 5.
(c) In going from a less dense to a more dense medium (the front of the film), the
reflected light has its phase inverted (a phase difference of ) relative to the
incident light. In going from a more dense medium to a less dense medium (the
rear of the film) the reflected light is in phase with the incident light.
(d) The phase difference between the light reflected from the rear of the film and the
front of the film is
 
2t
+ ,
= 2

and as t 0 the two reflections destructively interfere.

Problem OpaTh-FS01-4 from Courses-SC.

Numerical values
Give approximate numerical values for the following (be sure to give units):

(a) The binding energy of a nucleon in a typical nucleus.


(b) The binding energy of an electron in a hydrogen atom.
(c) The lifetime of a free neutron.
(d) The specific heat of Cu at room temperature (assumed greater than the
Debye temp.)
(e) The energy of a photon of light within the visible part of the spectrum.
Solution
(a) The binding energy of a typical nucleon is on the order of MeV.
(b) The binding energy of an electron within a hydrogen atom is 13.6 eV (Bohr
model of the atom).
(c) The lifetime of a free neutron is 15 minutes. This is roughly the time it takes
for the conversion of a down quark to an up quark through the weak interaction.
(d) The specific heat of Cu at room temperature is 3R (Dulong-Petit law)
(e) The energy of a photon of light within the visible part of the EM spectrum is
eV (E = hf = hc/, with 600 nm).

Problem 2 from Qualifying exam 2007-Fall.

10

Reaction and decay

Consider the reaction

p+pp+p++

(1)

Assume that the masses of proton and are 1 GeV/c2 .


(a) Consider a laboratory frame in which one of the two initial state protons
is at rest. What is the threshold energy, i.e. the minimum energy
that the incident proton must have for the reaction to be kinematically
possible? (Hint: What does this mean in terms of final state momenta in
the center-of-mass frame?)
(b) For the reaction at threshold as described in part (a), what is the mean
distance that the travels before it decays? (the mean lifetime of the
in its rest frame is 2.6 1010 s)
(c) What is the probability that at least one of the two s travels the distance
determined in (b)?
Solution
(Using units such that c = 1.) Let m, Ei , pi be the mass, energy, momentum of the
incident proton and p , E the momentum and energy of the , both in the laboratory
frame. Let (Ecms , pcms ) be the total energy of the system in the center-of-mass frame,
and (Elab , plab ) be the total energy of the system in the laboratory frame.
(a) By definition, at threshold the initial energy is just enough to produce the mass
energy of the new particles in the final state, i.e. all final state momenta in
the center-of-mass frame are 0. Therefore the total energy in the center-of-mass
frame is:
X
Ecms =
mi = 4m and pcms = 0.
(2)
The total energy and momentum in the laboratory frame is = energy/momentum
of incident proton (Ei , pi ) + energy/momentum of target proton (m, 0):
q
Elab = Ei + m = p2i + m2 + m, plab = pi .
(3)
To get the relation between Ecms and the lab energy, we use energy momentum
conservation and the fact that the square of the 4-momentum is Lorenz=invariant.
2
2
Ecms
p2cms = Elab
p2lab

(4)

2
Ecms
p2cms

Since

2
Elab

p2lab

(4m)2

(5)

2m(Ei + m)

(6)

Problem 5 from Qualifying exam 2007-spring.

we therefore have (4m)2 = 2m(Ei + m) and thus we get for the energy and
momentum of the incident proton at threshold:
Ei = 7m

(7)

and
pi =

Ei2 m2 = 4m 3

(8)

(b) Since in the center-of-mass frame all final state particles have 0 momentum, and
since by assumption they all have the same rest mass, they also have equal
momentum in the lab, i.e. 4p = pi or
q

p = pi /4 = m 3, E = p2 + m2 = 2m.
(9)
To calculate the mean distance traveled, we need the speed of . We use the
relation
E = m, p = m

(10)

where
=

v
1
, = p
.
c
1 2

(11)

The speed of is given by c , where


=

p
3
=
.
E
2

(12)

Because of time dilation, the mean lifetime of the s in the lab lab = .
The distance traveled by in this time is d = v lab ,

3
d=
3 108 2 2.6 1010 m 13.5cm
(13)
2
(c) The decay follows an exponential decay law: the number N (t) not yet decayed
at time t is N (t) = N0 et/ . In other words, the survival probability, i.e. the
probability that a particles time of decay tdecay is longer than some given time
td is
P (tdecay td ) = etd /tau

(14)

For td = , this gives e 0.368. Thus the probability that (1) has a lifetime
longer than its mean lifetime is P (1) = 1/e, and for (2) it is P (2) = 1/e. Since
the decays of the two particles are independent events, the probability for at
least one of the two (either of the two) surviving beyond its mean lifetime is
P (1 or 2) = P (1) + P (2) P (1 and 2).

(15)

The reason we have to subtract P (1 and 2) is to avoid double counting, since


P (1) includes those cases where in addition to particle (1) also particle (2) lived
beyond its mean time.
Thus the probability that at least one of the two s will travel the distance
determined in (b) is
P (at least one) = 2e1 e2 0.60.

(16)

Two nucleon system

The nucleons, proton and neutron, are spin 1/2 particles. Consider a twonucleon system with a relative angular momentum L and with a total spin S.
(a) For each of the following values of the total angular momentum (J = L+S)
and parity: 0+ , 0 , 1+ , 1 , 2+ , and 2 , list all possible combinations of
L and S.
(b) From part (a) list the configurations (L, S, J) that are allowed for identical
particles.
(c) The interaction between proton and neutron is identical to that between
the two neutrons and the potential is very shallow. Given that only proton
and neutron have a bound state (called deuteron) and the neutronneutron
system is unbound, what can you say about the spin and parity of the
deuteron? Justify your answer.
Solution
(a) Let us use the spectroscopic notation
urations:
0+ 1 S0
0 3 P0
1+ 3 S1 , 3 D1
1 1 P1 , 3 P1
2+ 1 D2 , 3 D2
2 3 P2 , 3 F2 .

2S+1

LJ . Below is the list of allowed config-

(b) For identical particles the total wave function must be antisymmetric. The spin
part is symmetric for S = 1 and antisymmetric for S = 0. The orbital part is
symmetric if L is even and antisymmetric if L is odd. Thus, the following states
are allowed for identical particles: 1 S0 , 3 P0 , 3 P1 , 1 D2 , 3 P2 , 3 F2 .
(c) The deuteron must be a protonneutron state that is not allowed for identical
particles; otherwise a bound neutron-neutron system would exist as well. This
means deuteron is in 3 S1 , 3 P1 , 1 P1 or 3 D2 as follows from (a) and (b). Due to
the centrifugal potential we must look for a state with the lowest value of the
angular momentum L. Therefore deuteron must be in 3 S1 state, implying it is
a spin-1 parity even bound state, 1+ . It can be a mixture of 3 S1 and 3 D1 but
due to centrifugal barrier it is predominately an S state.

Excited nucleus

The excitation energy of a free, excited nucleus at rest in the lab is E and the
mass of the excited nucleus is m. The nucleus de-excites by emitting a photon.
(a) Determine the frequency of the photon as measured in the lab, accounting
for nuclear recoil. Express your answer in the form
= a(E) + b(E)2

(1)

and find a and b.


(b) Explain in one sentence the sign of b.
(c) How does your answer change if the nucleus is rigidly attached to a crystalline lattice? Specifically, find a and b in this case.
Solution
(a) Conservation of momentum gives uspN = p
Momentum of the nucleus, pN ispN = mvr
where the subscript r refers to recoil.
Momentum of the photon, p isp =

h
E
=
c
c

where = E /h is the frequency of the emiited gamma ray. Hence, we have


mvr =

h
c

Conservation of energy gives usE = h +

mvr2
2

The frequency corresponding to E (resonant frequency, 0 ) isE = h0


The resonant frequency(in the frame of the nucleus) 0 is red-shifted to the
observed frequency (lab frame) by the recoil of the nucleus (Doppler effect)r
r
c vr
E c vr
= 0
=
c + vr
h
c + vr
Problem ModernPhysics1-2 from 2008Comps.

Hence, we can write the conservation of momentum in terms of the resonant


energy, Er
h E c vr
mvr =
c h
c + vr
E 2 c vr
2
vr = (
)
mc c + vr
We substitute vr2 in the expression for conservation of energy and with some
trivial rearrangementE
c vr (E)2
=
(
)
h
c + vr 2mc2
Comparing with the equation presented to us, we identify a and b asa=
b=

1
h

1
c vr
(
)
2mc2 c + vr

(b) b is negative, owing to the energy lost in the recoil of the nucleus. The Doppler
shift reduces the energy lost by a factor depending on the recoil velocity of the
nucleus.
(c) If the nucleus is rigidly attached to a crystalline lattice, the recoil acts on the
entire crystal due to the periodic arrangement of atoms in the crystal. The
recoil velocity is significantly killed owing to the large mass of the crystal. The
energy lost in the recoil process appears as vr2 , hence the emitted photon is much
closer to the resonant frequency. Mathematically, a remains the same, b which
represents a measure of the shift from resonance approaches zero.
These calculations form the basis of Mossbauer effect where the recoilless emission of gamma rays from a nucleus embedded in a crystal results in resonant
absorption. Gravitational red shift was verified using Mossbauer spectroscopy.

Two rockets

At time T=0,Rocket A leaves the earth with a velocity of c/2 in the x-direction
and Rocket B leaves the Earth with a velocity of -2c/3 in the y-direction.Assume
the earth is an inertial frame.Rocket A travels a distance of 6 light years and
stops.Call the stopping of Rocket A event 1.Rocket B travels a distance of 4
light years relative to the earths frame and stops.Call the stopping of Rocket
B event 2.Both rockets were traveling with constant velocity before stopping.
(a) Are events 1 and 2 time-like,light-like,or space-like separated? You must
explain your answer to receive credit.
(b) An observer in another reference frame measures a time interval between
the event 2 and event 1 of 3 years.What is the distance(in light years)
between the event 1 and 2 according to an observer in this frame ?
(c) Consider the time intervals between the event 1 and event 2 recorded by
all possible inertial observers.There is an inertial frame that gives the
minimum time interval.What is this minimum time interval ?
(d) There is an inertial frame that gives the minimum spatial separation between event 1 and event 2. What is this minimum spatial separation ?
(e) Consider an inertial frame that moves along the direction of the separations
between event 1 and event 2 as viewed in the Earths reference frame. Find
the velocity vector (relative to Earth) for the frame that gives the shortest
possible distance between events 1 and 2.
Solution
(a)
s2 = (ct)2 x2 y 2 z 2

(2)

t = t1 t2

(3)

12
6ly
=
ly
t1 =
c/2
c
4ly
6
t2 =
= ly
2c/3
c
6
t = ly
c
s2 = (6ly)2 (6ly)2 (4ly)2 = 16ly 2
since s2 is less then zero,events 1 and 2 have to be space-like separated.
Problem MP06 from 2008Comps.

(4)
(5)
(6)
(7)

(b) The distance is


t0 = 3y

(8)

ct0 = 3ly

(9)

s = 16ly = (3ly) r
2

r = 25ly

(10)
(11)

r = 5ly

(12)

(c) Since the two events are space like separated , there must exist a frame where the
two events occur at the same time but at different locations.Therefore, t0 = 0.
(d) The spatial separation is
s2 = (ct)2 r2 = 16ly 2
0

t = 0

(13)
(14)

r2 = 16ly 2

(15)

r = 4ly

(16)

(e)
r0 = (r ct)

(17)

r)
c

(18)

t0 = (t

using the assumption that the frame found in parts (c) and (d) travels along the
distance between event 1 and 2
6

0 = ( ly
52ly)
c
c
6
=
52
6
v= c
52

(19)
(20)
(21)

K decay
Suppose K 0 is created with a total energy 10 GeV in the lab.

(a) How far does K 0 travel in the lab


(b) Find the smallest possible Kinetic Energy.
(c) Find the angle of decay.
Solution
(a) For this problem we can use the standard Lorentz transformation:
t = t0 , Where =

r 1
2
1 v2
c

And we can use the Energy equation:


E = m0 c2
From the energy equation and solving for we find:
=

ELab
m0 c2

10000
498

Therefore, the answer to how long the particle exists in the lab frame is:
tLab = trest 20 1010 [sec]
The velocity of the Kaon is:
v=

1
2

0.99876 c

Therefore the final answer is:


d = vt = 0.99876 20 1010 (3x108 ) 0.599[meters]
(b) In the center of mass frame, the resulting velocity of each 0 is:
r
m2
E = m0 c2 = v = 1 4 m2 0.84027 c
K

Using the velocity addition formula, we can find the velocity difference between
the lab frame and the frame of the pion. The two velocities are in opposite
directions.
s=

uv
1 uv
2

0.9858 c 5.956

KE = m c2 ( 1) 675[M eV ]
(c) If the two pions have equal energy in the lab frame then the pions decayed in a
plane perpendicular to the velocity of the original kaon. Therefore the decay
angle is defined by the four-momentums.
(P,K )2 = (P,1 + P,2 )2 m2K = 2m2p + 2(E2 p1 p2 cos )
Solving for
Problem problem13modern from 2008Comps.

= arccos(

m2

E
K
m2
+ 4 2
p1 p2

p1 p2 = p2 = E4 m2
E is the original energy in the lab frame of the kaon.
theref ore 4.6degrees

Cosmic ray

Cosmic ray photons from space are bombarding your laboratory and smashing
massive objects to pieces. Your detectors indicate that two fragments, each of
mass m0 , depart such a collision moving at a speed of 0.6c at angles of 60
relative to the photons original direction of motion.
(a) In terms of m0 and c, what is the energy of the cosmic ray photon?
(b) In terms of m0 , what is the mass M of the particle being struck (assumed
to be initially stationary)?
Solution
(a) The components of the fragments momenta in the direction perpendicular to the
original photons direction of motion will cancel, and the components along that
direction will reinforce. Conservation of three momentum tells us that
pi = 2 pf cos(60o ).
Using the relativistic definition of momentum, pf = m(vf )vf , where (vf ) =
q
1/ 1 vf2 /c2 = 1.25,
p = pi = 2 pf cos(60o ) =

2 m0 v cos(60o )
p
.
1 v 2 /c2

Now since for a photon E = p c, then


E = 2 m0 0.6c2 cos(60o ) 1.25 =

3
m0 c2 .
4

(b) Use conservation of relativistic energy, and the fact that the total relativistic
energy (including rest energy) of the final fragments is
Ef = m0 (vf )c2 = 1.25 m0 c2
so that
Ei = E + M c2 = 2Ef ,
and we have that
M=

2Ef E
2.5 m0 c2 0.75 m0 c2
=
= 1.75 m0 .
2
c
c2

Problem ModQM-F01 from 2003-Proficiency-Spring.

Cosmic ray collision

A cosmic ray photon strikes a stationary particle of mass M, which fragments


into two pieces, each of mass m0 . The two pieces depart the collision moving at
a speed 53 c at 60 to the photons original direction of motion. (c is the speed of
light in vacuum.)
(a) Determine the energy of the cosmic ray photon in terms of m0 and c.
(b) Determine M ,the mass of the particle being struck.
Solution


pr c
M c2
pr 0


0 + 0
0
0

q
(m0 c2 )2 + p2 c2

p cos 60
=

p sin 60
0

pr = p = q

m0 v
1


v 2
c

q
(m0 c2 )2 + p2 c2


p cos 60
+

p sin 60
0

m0 c
v
q
2
c
1 vc

(a)
Er = pc =

v
m0 c2
3
1
q
 = 5q
c
v 2
1 c
1
Er =

3
m0 c2
4


3 2
5

m0 c2

(b)
s

 2
3
3
m20 c4 m0 c2
4
4


5
3
7
M = 2
m0 = m0
3
4
4

Mc = 2

(m0 c2 )2 +

Concepts
Provide a brief qualitative description for each item listed below.

(a) Heisenberg uncertainty principle.


(b) The fundamental forces.
(c) Michelson-Morley experiment.
Solution
(a) The Heisenberg uncertainty principle states that it is impossible to know simultaneously the precise position and momentum, or time coordinate and energy,
of a particle. The product of the uncertainties of the two measurements must
be at least h
/2. An example of the energy-time relation is the product of the
lifetime of an unstable state and the uncertainty in the energy of the photon it
emits (the decay line-width).
(b)In order of increasing strength these are: the gravitational force between any
two massive objects, which falls off like 1/r2 and so has infinite range; the
electromagnetic force between any two charged particles, which has the same
force law and infinite range; the weak force responsible for, for example, the beta
decay of a neutron into a proton, and electron and an anti-electron-neutrino; and
the strong force which is responsible for binding quarks together into protons and
neutrons and also for the nuclear force between nucleons. The weak force has
extremely short range because of the large mass of the force carriers (the W and
Z bosons) and the strong force has effectively a short range because confinement
of colored objects requires any final-state particles in an interaction to be color
neutral.
(c) This experiment was set up to determine how quickly the Earth moved through
the aether which was thought to be the medium in which light propagated. If
light moved with a fixed speed through the aether, the apparent speed of a
beam of light through an apparatus fixed to the moving Earth would depend
on whether it moved perpendicular or parallel to the direction of motion of the
Earth. A beam of light was split by a half-silvered mirror and sent along two
paths at right angles to each other, reflected back by mirrors and then recombined. By counting the number of interference fringes moving by an observer as
the apparatus was rotated by 90 , Michelson and Morley planned to measure the
difference in speed of the two beams. They saw no such motion of the fringes,
which invalidated the idea of the aether.

Problem ModQM-F99-2 from 2003-Proficiency-Spring.

Photoelectric effect

When light of wavelength 520 nm is incident on the surface of a metal, electrons


are ejected with a maximum speed of 1.78105 m/s. What wavelength is needed
to give a maximum speed of 4.81 105 m/s?
Solution
The electrons are bound to the metal with a work function , which is the minimum
energy required to remove an electron from the metal. Assuming an electron which
requires only this minimum energy, the energy h of the photon goes into releasing
the electron from the surface and any extra energy is kinetic energy of the electron,
so that (note the electron velocity is non relativistic)
h =

hc
1
2
= + me vmax
.

For two different wavelengths we have


hc
1
hc
2

=
=

1
me v12
2
1
+ me v22 ,
2
+

where v1 and v2 are understood to be maximum velocities, and so we can eliminate


to obtain
hc
1
me v12
1
2
hc
2

=
=

hc
1
me v22
2
2
hc
1
+ me (v22 v12 )
1
2

or
2

=
=
=

hc
hc/1 + 12 me (v22 v12 )
(
)
[4.81 105 ]2 [1.78 105 ]2
1
1240 eV nm
6
+ 0.511 10 eV
1240 eV nm/
520 nm
2
(3 108 )2
420 nm.

Problem ModQM-F99-3 from 2003-Proficiency-Spring.

Concepts 2
Provide a brief qualitative description for each item listed below.

(a) Cerenkov
radiation.
(b) Plancks constant.
(c) Rutherford scattering
Solution

(a) Cerenkov
radiation occurs when a charged particle travels through a material
faster than the speed of light c/n in that material. The radiation is emitted in
a forward opening cone whose opening angle depends on the particles speed. It
is like the shock wave which occurs when a body moves faster than the speed of
sound in air.
(b) Plancks constant h relates the energy of a photon to the frequency of the electromagnetic radiation, E = h. As such, it provides the link between the wave
and particle nature of the photon (and other particles). Planck introduced h to
explain the failure of the Rayleigh-Jeans formula for black-body radiation, which
led to an infinity (the ultraviolet catastrophe) in the amount of energy emitted
by a black-body when high energy photons were considered. His suggestion was
that atoms emit energy only in discrete bundles whose energy was proportional
to the photon frequency.
(c) Rutherford scattered alpha particles (4 He nuclei) off a gold foil. It was expected
that there would be few backward scattering events because it was thought that
the material in atoms was roughly evenly spread out. In fact the distribution
of the scattering angle showed the existence of small massive centers of positive
charge in the gold foil. This led to the Rutherford scattering formula (for point
like charged particles) and to the Bohr model of the atom.

Problem ModQM-F99-5 from 2003-Proficiency-Spring.

10

Muon

(a) How fast does a muon have to travel to have the same energy as a charged
pion at rest?
(b) What is the de Broglie wavelength of the muon?
(c) If you could measure the momentum of the muon with 10% accuracy, how
well could you measure its position along its direction of motion?
Solution
(a) The mass of the muon is m = 106 MeV/c2 , and that of the pion is m = 140
MeV/c2 . The (total relativistic) energy of the moving muon is therefore
m c2 = m c2 ,
so that
=

m
,
m

and so
v2
1 2
c

v
c

2

1
m
=
2
m
s

2
m
1
= 0.65.
m

(b) The de Broglie wavelength is given by


=

h
hc
=
.
p
pc

Now the momentum of a relativistic particle is p = mv, so we have


p
p
hc 1 (v/c)2
1240 MeV fm 1 (0.65)2
hc
hc
=
=
=
=
= 13.7 fm.
pc
mvc
mc2 (v/c)
106 MeV 0.65
(c) According to the uncertainty principle we have that
px

h
h
=
.
2
4

This means that


x

h
h

=
=
= 10.9 fm.
4(0.1p)
4(0.1h/)
0.4

Problem ModQM-F99-7 from 2003-Proficiency-Spring.

Muon decay

Radioactive decay proceeds according to the very simple law that the probability
of decay for any given system is constant, that is
1 dN
= .
N dt

(1)

(a) Derive an equation for N (t), the number of particles remaining as a function
of time, in terms of N0 the number at time t = 0, the time t, and the decay
constant .
(b) In addition to , radioactive decay is often described in terms of the half life
t1/2 or the mean life at which times N (t) = 1/2N0 or N (t) = (1/e)N0 ,
respectively. Derive expressions for t1/2 and in terms of as well as a
relation between t1/2 and .
(c) The mean life of the muon is = 2.2 s in its rest frame (proper time).
Assume a group of muons is produced 8000 m above the surface of the
earth and all travel directly downward at a speed of 0.995 c. (Use c =
3108 m/s). For this part, ignore special relativity. What is the flight time
to the surface of the earth classically and what is the surviving fraction
(N (t)/N0 ) of muons?
(d) Now consider the proper treatment using special relativity and the Lorentz
transformation from the point of view of an observer traveling with the
muons. In this frame the mean lifetime is unchanged at = 2.2 s. What
distance does this observer perceive as the distance to the earths surface
and what is the travel time? Calculate the surviving fraction now.
(e) Lastly consider what an observer on earth would perceive in special relativity. This observer would see that the muons must travel 8000m. What
mean life would the earth based person observe and what would be the
travel time to the surface? Calculate the surviving fraction that the earth
based person would observe.
(f) Comment on the implications of parts (c) - (e) for a physicist on the ground
detecting muons. What does the principle of relativity and the independence of the frame of reference on physical outcomes say about what the
relation between the answers for (d) and (e) should be?
Solution
(a) The decay is governed by the following differential equation
dN
= dt,
N

thus

N (t) = N0 et.

(2)

(b)
N (t1/2 ) = N0 et1/2 =

N0
2

t1/2 =

ln(2)
.

(3)

N ( ) = N0 e =
=

N
e

1
.

ln(2)
1
=
t1/2 = ln(2).

t1/2

(4)
(5)

(c)
t = L/v =

8000m
= 26.8s.
0.995 3 108 m/s

(6)

N (t)/N 0 = et/ = e26.8/2.2 = 5.12 106


(d)
1
= p
= 10
1 v 2 /c2
L
= 8000m/10 = 800m

L
800m
t=
=
= 2.68s.
v
0.995 3 108 m/s
L=

N (t)/N 0 = et/ = e2.68/2.2 = 0.30


(e) t = 26.8s as in (c), but = = 10 2.2s = 22s. N (t)/N0 = et/ =
e26.8/22 = 0.30.
(f) Classical physics would predict a very tiny fraction of parts per million muons
surviving to the ground, whereas special relativity predicts 5 orders of magnitude greater survival probability of 30% similar to what is observed. Also the
observers on the ground and traveling with the muon both predict the same
survival probability.
One might wonder about the effect of general relativity due to Earths gravity. It is
quite small. One can solve this problem exactly using the Schwarzschild metric


1

2GM
2GM
2
dt

dr2 .
(7)
ds2 = c2 dt20 = 1
rc2
rc2
p
By setting M = 0 one immediately recovers t0 = t (1 v 2 /c2 ) which corresponds to
the above result. The correction from the gravity is very small
2GM
2gr
= 2 109
rc2
c
for all distances ranging from the surface r = 6400km to 8km above the surface.

Thermodynamic cycle 2

Consider the following process done on an ideal monatomic gas. 1000 joules
of heat are added at constant pressure. The pressure is 1 atm. The system
expands in the process and its temperature rises 10 K.
(a) Find the number of moles present.
(b) Find the increase in volume V .
(c) Find the entropy change in terms of the initial temperature T0 .
Solution
(a) According to first Law of Thermodynamics Q = U + P V . For an ideal
gas P V = nRT and for a constant pressure process P V = nRT . For a
monoatomic ideal gas U = (3/2)nRT , thus
Q = (3/2)nRT + nRT = (5/2)nRT
n = Q/((5/2)RT ) = 1000J/((5/2) 8.31J/(mole K) 10K)) = 4.8moles
(b) P V = Q U , and
U = 3/2nRT = (3/2)RT Q/((5/2)RT ) = 3Q/5
Thus P V = Q (3/5)Q = (2/5)Q and
V = (2/5)Q/P = (2/5)1000J/(1.013 105 P a) = 3.95 103 m3
(c) T S = Q = (5/2)nRT for the constant pressure process. S = (5/2)nRT /T ,
Integrating both sides


T0 + 10K
Q
ln
S = (5/2)nR ln T /T0 =
T
T0


T0 + 10K
= 100 ln
T0

Problem 1 from Qualifying exam 2011-Spring.

4-momentum conservation

An energetic photon hits a stationary particle of rest mass M which breaks


into two equal mass particles of rest mass m0 each. The two particles leave the
collision with equal velocities 0.6c at equal angles of 60 degrees relative to the
direction of the incoming photon. Express all your answers below in terms of
m0 and c.
(a) What is the magnitude of the relativistic momentum of one of the outgoing
particles?
(b) Use relativistic momentum conservation to find the magnitude of the momentum of the incoming photon.
(c) What is the energy of the incoming photon?
(d) What is the total (including rest mass) energy of one of the outgoing
particles?
(e) Use relativistic energy conservation to find the rest mass M of the struck
particle.
Solution
(a)
p
3
= 1/ 1 v 2 /c2 = 5/4, pf = m0 0.6c = m0 c
4
(b) Momenta of outgoing particles perpendicular to photon direction must add to 0,
momenta in direction of photon must add to equal its value:
p = 2pf cos(60 ) =

3
m0 c
4

(c)
E = p c =

3
m0 c2
4

(d)
Ef = m0 c2 =

5
m0 c2
4

(e)
E + M c2 = 2Ef , M = (2Ef E )/c2 =

Problem 1 from Qualifying exam 2012-Spring.

7
m0 .
4

Pb nucleus

Using Einsteins mass-energy relation, calculate the fraction of the mass of the
Pb (Z = 82) nucleus that can be attributed to the Coulomb interaction. For
this estimate assume that the lead nucleus is a charged sphere of radius R =
7fm = 7 1015 m. The charge is uniformly distributed throughout the volume
of the sphere. Assume that the total nuclear mass is 1 amu=931MeV/c2 =
1.66 1027 kg per each nucleon.

208

(a) What is the electric field, both outside and inside the sphere?
(b) What is the electrostatic potential at all points in space?
(c) What is the electrostatic energy of this uniformly charged sphere?
(d) Compute the 208 Pb nuclear mass and the Coulomb energy from part (c),
and find the fraction of the mass due to the Coulomb interaction.
Constants: e = 1.61019 C; k = 1/(40 ) = 8.9109 N m2 /C 2 ; = 1/137; h
c =
197MeV fm; c = 3 108 m/s.
Solution
(a) Let Q be the total charge of the nucleus: Q = Ze. Using Gauss law we find
~ =
E

1 Q
~r, (r R)
40 r3

~ =
E

1 Q
~r, (r < R)
40 R3

(b) The electrostatic potential at any point p is given as an integral


Z p
Z
~ d~r =
~ d~r
(p) =
E
E

This form assures that the potential is zero at infinity. We separate regions of
integration 1-interior, 2-exterior
Z R
Z
~ 1 d~r +
~ 2 d~r
(p) =
E
E
p

This results in:

r2
1 Q
(3 2 ), (r R)
80 R
R
1 Q
2 =
, (r > R)
40 r

1 =

(c) The energy can be found as


Z
Z R
1
3 1 Q2
3 1 Z 2 e2
U=
d3 r(r)(r) = 2
1 (r)(r)r2 dr =
=
2
5 40 R
5 40 R
0
Problem 5 from Qualifying exam 2012-Spring.

(d) The Coulomb energy of the led nucleus using =


U=

e2
40 h
c

3
Z2
3 1
822

hc
=
197MeV fm
= 829MeV
5
R
5 137
7fm

The mass of the lead nucleus is 208 times 1amu which means that the Coulomb
energy is about 0.4% of the mass.
f=

829MeV
= 0.43%
208(931MeV/c2 ) c2

Proton in 1d well

A proton (mass 938 M eV /c2 ) is in a 1-dimensional square well 100 nm in


length, with walls that are infinitely high. The center of the well is at x = 0.
(a) Is the proton relativistic?
(b) What is the expression for the protons allowed energy levels? Calculate
numerically the energy of the ground level.
(c) Calculate the expression
of the expectation value of the position of the


proton squeared x2 in the ground level.
Solution
(a) Lets consider the uncertainty principle,
px

h
2

(8)

Thus,
p

hc

=
2x
2xc

Since a proton is confined in a 100nm 1-dimensional square well, x = 100nm.


Also, h
c = 197.33M eV f m. Therefore,
pc

=
=
=

=
=

(mp vc)


M eV
938 2 vc
c

v
938M eV
c
hc

2xc
197.33M eV f m
2 100nm
0.99eV

Therefore,

v
1.05 109
c
This means that the protons speed iv much smaller than the speed of light. So,
the proton is not relativistic.

(b) Since V (x) = 0 for L2 < x <

L
,
2

the Shroedinger equation is

2 2
h
= E.
2m x2

The solutions for this equation is: will have the form:
(x) = A sin(kx) + B cos(kx),
Problem MP17 from FSU Qualifying Exam Wiki.

where k =

p
2mE/
h2 . Considering the boundary conditions, we have:
(


A sin kn x L2 , L2 < x <
n (x) =
0,
others
q
2
where the normalization constant A is
and
L
n
kn =
=
L

L
2

2mEn
.
h2

Therefore,
n2 2
h2
2mp L2
The ground state energy will be for n = 1
En =

E1

2 2
h
2mp L2

2 c2 2
h
2mp c2 L2

(197.33M eV f m)2 2
2(938M eV )(100nm)2

2.05 108 eV

(c) The ground state wave function is:


r
1 (x) =

 

2
x
1
sin

L
L
2

Therefore,

2
x

=
=
=
=
=

2
L

2
L

2
L

L
2

L
2
L
2

1 x2 1 dx
x2 sin2

L
2
L
2


x

L
2


dx

 
x dx
L
L
2

Z L

2
2
2 1 + cos 2 L x
x
dx
L L
2
2
 3
Z L
 
2
2
L
1
+
x2 cos 2 x dx
3L 2
L L
L
x2 cos2

Using the formula

x2 cos axdx =

2x
a2

cos ax +

x2
a

2
a3

sin ax we can solve the

integral

x2

2
3L

L2
12

L2
2

 3
Z L
 
2
L
1
x2 cos 2 x dx
+
2
L L
L
2
 2


L
Lx
1 L2 x
2x
L3
2x 2
+
cos
+
3 sin
L 2 2
L
2
4
L L
2


1
1
2 .
6

Lepton in magnetic field

A -lepton moving at relativistic speed v is produced in a proton-proton


collision. The -lepton moves in a 1.0T magnetic field such that the angle
between B and v is 85 . As viewed along the direction of B, the projected path
of the -lepton is a circle of radius 10m.
(a) What is the momentum (in eV/c) and the Lorentz factor of the -lepton?
(b) In the rest frame, the -lepton has a mean lifetime = 2.197 106 s.
In the laboratory frame, what is its mean lifetime? (For this calculation,
ignore the acceleration of the -lepton).
(c) Describe the trajectory (either qualitatively or quantitatively) of the leptons path. How long will it take for the -lepton to travel one complete
orbit in the plane perpendicular to the magnetic field? In this time, how
far will it move in the direction of the magnetic field?
Solution
(a) The magnetic force will work like a centripetal force, then we can match the both
forces as:
~m
F

~
q~v B

qvB sin 85

qBr sin 85

~c
F
v2
r
r
2
v
m0
r
m0 v
m0

The last expression is precisely the momentum p = m0 v, where m0 is the rest


mass of the lepton, therefore:

m0 v

qBr sin 85

(e)(1T )(10m) sin 85


!

2.998 108 m
s 
s
sin 85
(e) 1V 2 (10m)
m
c

=
=
=


eV
2.998 109 sin 85
c
9 eV
2.987 10
c

From the definition of the momentum, we can solve for the velocity v:
Problem MP12 from FSU Qualifying Exam Wiki.

p
v
p
1 v 2 /c2

m0 v
p
m0
2
 2
m0
1
+
p2
c2

0.106 109 eV
c2

2

2.987 109 eV
c2

c2

!2

2 + 1

0.997c

(9)

Therefore,
= r

v2
c2

= 12.92.

(b) We need to calculate the dilatation of the time for the laboratory frame:
0


(12.92) 2.197 106 s

28.38 106 s

(c) The trajectory will be like a curl. In the plane perpendicular to the magnetic
field, it will be a circular movement, and in the parallel direction to the magnetic
field, it will be a movement with constant velocity. The time for one loop will
be:

2r
v

2 (10m)

(0.997) 3 108 m
s

0.23 106 s

The travel distance in this time in the direction parallel to the magnetic field
will be:

=
=
=

(v cos 85 ) t


m
cos 85 0.23 106 s
(0.997) 3 108
s
5.94m

Bohr model

(a) Calculate the frequency of revolution and the orbit radius of the electron
in the Bohr model of hydrogen for n = 100, 1000, and 10,000.
(b) Calculate the photon frequency for transitions from the n to n-1 states
for the same values of n as in part (a) and compare with the revolution
frequencies found in part (a).
(c) Explain how your results verify the correspondence principle.
Solution
(a) Well start by calculating the Bohr radius, which we can derive by Newtons
second law and the Bohr quantization rule for angular momentum:
F = me ar = me
F =

v2
r

e2
40 r2

v2
e2
e2
=
me v 2 =
r
40 r
40 r
n
h
L = r p = me rv = n
hv=
me r
n2
h2
n2
h2
me v 2 = me 2 2 =
me r
me r2
2 2
2
n
h
e
=
me r2
40 r
40 n2
h2
rn = 2
e
me
r100 = .5293m
me

r1000 = 52.93m
r10000 = 5290.m
The radii for n = 10000 and n = 1000 would be easily visible to the naked eye.
The radius for n = 100 would be easily visible with a weak microscope.
It is unclear what theyre looking for when asking for frequency of revolution.
They could be asking for either (calculated here) or , which are related by
= 2.
n
h
v
= =
r
me r2
Using the radii found above we have:
100 = 4.13 1010 Hz
1000 = 4.13 107 Hz
10000 = 4.13 104 Hz
Problem Mod13 from FSU Qualifying Exam Wiki.

(b) The frequency of the photon will correspond to the change in energy between the
two states. In case you dont remember the equation for this energy it can be
derived from the total energy of the electron:
E=

1
1 e2
me v 2
2
40 r

1 e2
1 e2
1 e2

=
2 40 r
40 r
2 40 r
 2 2
2 2
1
h
me
e
40 n
E= 2
r= 2
e
me
40
n2
2
h
E=

R
n2
where R is the Rydberg constant: R = 13.6eV .
So the energy of the photon is given by change in energy between states:
!
1
1
h = R
2
n2f
ni
E=

R
=
h

1
1
2
n2f
ni

Rc
=
2
hc

1
1
2
n2f
ni

100 = 6.68 109 Hz


1000 = 6.59 106 Hz
10000 = 6.582 103 Hz
In order to compare these values to the values obtained in part (a) we need
to turn ordinary frequency into angular frequency: = 2 Below lists the
angular frequency from part (a) and corresponding angular frequency for part
(b):
(a) 100 = 4.131010 Hz (a) 1000 = 4.13107 Hz (a) 10000 = 4.13104 Hz
(b) 100 = 4.201010 Hz (b) 1000 = 4.14107 Hz (b) 10000 = 4.13104 Hz
(c) The correspondence principle (formulated by Bohr) states that quantum mechanical systems reproduce classical results in the limit of large quantum numbers.
As you can see above the frequency of the emitted radiation in the classical limit
(b) is equal to the frequency of rotation of the electron around the nucleus (b)
where we used the quantum principle of quantization of angular momentum.
These numbers agree better with larger quantum number n.

Uncertainty principle
Use the Heisenberg uncertainty principle to show that

(a) electrons are unlikely to be constituents of atomic nuclei


(b) their existence within atoms is reasonable.
Solution
The Heisenberg uncertainty principle states
xp >

h
2

We can write the energy as:


E=

p2
2m

The mass of the electron is:


m = 9.109 1031 kg
The size of a nucleus and an atom are approximately:
xnuc 1015 m
xatom 1010 m
Solving the uncertainty relation for p and substituting these values for x:
pnuc 5.27 1020 J s/m
patom 5.27 1025 J s/m
Substituting these values into the energy equation:
Eatom = 1.53 1019 J = 0.9eV
Enuc = 1.53 109 J = 9.5 109 eV = 9.5GeV
Nuclear binding energy is on the order of MeV and electron (atomic) binding energy
is on the order of eV. Thus an electron with energy on the order of GeV could not be
bound to the nucleus. However, and electron with energy on the order of eV could be
bound within an atom.

Problem Mod18 from FSU Qualifying Exam Wiki.

Magnetic moment of electron

Part I) Originally the electron was thought to be a tiny charged sphere (we
now treat it as a point object with no space extensions). Find the equatorial
speed assuming the electron is a uniform sphere of radius 3x106 nm. Compare
the answer with the speed of light.
Part II) A cylinder of radius R, with mass M uniformly distributed throughout
the volume. The angular speed, is about the longitudinal axis.
~
(a) Obtain L.
(b) If a charge Q is distributed uniformly over the curved surface only, find the
~ to deduce
magnetic moment
~ of the rotating cylinder. Compare
~ and L
the g-factor.
Solution
Part I
We know that the magnitude of angular momentum, L, is given by L = rp = rmv.
We also know that for electrons (which are spin 1/2) that the angular momentum is
equal to
h/2. Therefore L = mrv = h
/2.
Plug in the values for h
, the mass of the electron (.511 MeV/c2 ), and the radius of the
electron and you find that velocity is:
v = 2x1011 m/s, which is of course faster than the speed of light!!
Part II
Z
~ = ~rx~v d3 r, where is the density, ie = M/V .
(a) L
Write ~v as ~v =
~ x~r =
z x (r
r + z z) = r
Z
Z h


 
i
~ = (r
L
r + z z) x r d3 r =
r
r + r + r + z z d3 r
~ =
L

L/2
Z

Z2
d
0

L/2

ZR
dz



rdr r2
z zr
r . Note that the second term goes to

zero when you do the z integration.


L/2
Z
ZR
M R2 z
~ = M 2
z
dz r3 dr =
L
V
2
L/2

This solution is correct since L = I if the angular momentum vector is parallel


to the angular velocity vector and for a cylinder I = M R2 /2.
Z
~ 3r
(b)
~ = 1/2 ~rxJd

J~ = ~v ; where again, ~v =
~ x~r =
z x (r
r + z z) = r,
Q
and, =
(r R)(Z L/2)
2RL
Plugging in these values and performing the integral as before yields:
Problem Mod11 from FSU Qualifying Exam Wiki.


~ = QR2
z /2
gQ ~
~ and
To solve for the g-factor use:
~=
L and plug in the values for L
~
2M
found above. Surprisingly g = 2, which is approximately the g-factor for the
electron.

10

Short questions

The following questions are to be answered by a single number (order of


magnitude) or by brief single sentence statements.
(a) What is the ionization potential of the hydrogen atom (in eV)?
(b) What is the electrostatic energy (in eV) of two electronic charges spaced
1
A apart?
(c) A capacitor has two cracks in the dielectric as shown. At which of the two
cracks will breakdown occur first as the voltage is increased? Explain.

(d) Why is the sky blue and the sunset red?


(e) What is the range of wavelengths of visible light?
(f) What is the ratio of the magnetic moment of the electron and the proton?
(g) Are most substances (e.g. water, benzene) paramagnetic or diamagnetic?
Explain.
Solution
(a) 13.6eV;
(b)
(1.602 1019 )2
e2
=
= 14.4eV ;
40 r
4 3.1415 8.8542 1019 1 1010
(c) B breaks down first. Suppose E is the electric field inside the capacitor, then
EA = E (Et continuous), EB = E/1 = E (Dn continuous). Thus EB > EA
and B breaks down first.
(d) Rayleigh scattering k4 : the relative percentage of short waves such as blue
is heavier in the scattered lights, while in the forwarding lights, the relative
percentage of longer waves such as red is heavier.
-7800A
;
(e) 3900A
(f) e /n = mp /me 1837;
(g) Diamagnetic. Electrons gyrorate in the direction of externally applied field; since
they carry negative charges, the induced magnetic moments are in the opposite
direction.
Problem 1 from UCLA 1988Fall.

Two level system

The D0 are particle and anti-particle. The wave function that describes a
neutral D can be described as two component wave function.
 0 
 
 
D
1
0
0
0

|i =
+D
0 = D
D
0
1
0 i are the particle and antiparticle components. The time
where |D0 i and |D
evolution of such a wave function can usefully be written
i

|i = H|i = (M )|i
t
2

with the Hamiltonian



H=

M 2i

M12
2i 12

M12 2i 12
M 2i

M and are the 2 2 mass and decay matrices; CPT invariance requires
M11 = M22 = M and 11 = 22 = . Assumming CP invariance (which is
approximately true), M12 and 12 are real. In units where h
= c = 1,
(a) What are the eigenstates of this Hamiltonian (label them D1 and D2 )?
(b) What are the lifetimes (1 and 2 ) and the masses (m1 and m2 ) of the two
states? (Hint:Solve for the complex eigenvalues of the two states.)
(c) Assuming the D is produced as a particle, |(t = 0)i, calculate the proba 0 in the limit that M , 
bility as a function of time the particle is a D
and M t, t  1 where M |m1 m2 |, |1 2 |.
0 |i|2 maximum?
(d) At what time is |hD
Solution
(a)
(M

i
i
E)2 (M12 12 )2 = 0
2
2

Solving for E we have.


E = M

i
i
(M12 12 )
2
2

Now we are going to find the eigenstates



H=

M 2i

M12
2i 12

M12 2i 12
M 2i



a
b

Choosing a = 1 we have
b=1
Problem G3P from 2008Comps.




i
i
a
M (M12 12 )
b
2
2

Then the eigenstate is


1
|D1 i =
2

1
1

For the other eigenvalue we have


1
|D2 i =
2

1
1

(b) Looking at the eigenstates we have


1
( + 12 )
2
1
2 = ( 12 )
2

1 =

and for the masses

1
(M + M12 )
2
1
m2 = (M M12 )
2
(c) We can write |i ad a linear convination of |D1 i and |D2 i as
m1 =

1
1
|i = eiE+ t |D1 i + eiE t |D2 i
2
2
 0   1 iE t 1 iE t 
+

D
e
+ 2e
2
=
1 iE+ t
1 iE t
D0
e

e
2
2


D0
D0

"


=

i +M i
1 i(M 2
12 2 12 )t
e
2
i +M i
1 i(M 2
12 2 12 )t
e
2

i
i
+ 21 ei(M 2 M12 + 2 12 )t
i
i
21 ei(M 2 M12 + 2 12 )t

So the probability is
ih
i
1
1
1
1 h i(M +M12 )t 12 (+12 )t
e
P =
+ ei(M iM12 )t 2 (12 )t ei(M +M12 )t 2 (+12 )t + ei(M iM12 )t 2 (12 )t
4
After some algebra we get

1  (+12 )t
e
+ e[i(m1 m2 )]t + e[+i(m1 m2 )]t + e(12 )t
P =
4
Simplifying we have that the probability is given by
P =

1 t
e
[cosh (12 t) cos (M t)]
2

(d) Taking the derivative of the probability wih respect to time we have
P

1
= et [cosh (12 t) cos (M t)] + et [12 sinh (12 t) + M sin (M t)]
t
2
2
To find the maximum we do the derivative equal to zero. And since 12 t and M t
are small we can taylor expand this to second order, so we get

(12 + M ) t2 + (12 + M ) t = 0
2

Since t = 0 is the minimum the maximum should be


2
t=

Proton and antiproton

An antiproton with Kinetic Energy= 1.00GeV collides with a stationary proton to form a final state which is a new particle, X 0 . In the following please
give numerical answers correct to at least two significant figures. Protons and
antiprotons have rest-mass energy mp c2 = 0.94GeV .
(a) Calculate the rest-mass energy and velocity of the X 0 particle. You may
express the velocity in units of c.
(b) The X 0 subsequently decays into two photons. One of the photons is emitted
along the direction in which the X 0 was moving. Find its energy and the energy
and direction of the other photon.
(c) Another similarly produced X 0 decays as shown below, with the photons
emitted at equal angles, , to the direction in which the X 0 was moving. Find
the energies of the photons and the angle .
Solution
(a) Let T = 1.0GeV and c = 1 Using conservation of 4-momentum
(mp , ~0) + (Ep , p
~) = (EX , p
~x ).

(1)

Squaring both sides and using E 2 p2 = m2


m2p + m2p + 2Ep mp = m2x

(2)

rearranging and using Ep = T + mp we get


p
mx = 2mp (Ep + mp )
p
mx = 2mp (2mp + T )

(3)
(4)

Plugging in for mp and T .


p
mx = 2 0.94GeV (2 0.94GeV + 1.0GeV ) = 2.3GeV

(5)

And to determine the velocity we use


p
Ep2 m2p
px
p
p
v=
=
=
=
=
Ex
Ex
T + 2mp
T + 2mp

(T + mp )2 m2p
T + 2mp

(6)

Plugging in for mp and T .


q
v=

(1 + 0.94)2 0.942 GeV


= 0.59
1 + 2(0.94)
GeV

(7)

(b) Again using conservation of 4-momentum


(Ex , p~x ) = (p , p
~ ) + (p0 , p~0 ).

(8)

Rearranging and using the fact that the angle between p~x and p~ is 0
(Ex , p~x ) (p0 , p~0 ) = (p , p
~ ).
Problem G4P2 from 2008Comps.

(9)

Squaring and solving for p


m2x 2Ex p + 2p~x p~ = 0

(10)

m2x 2Ex p + 2px p cos00 = 0

(11)

m2x

2p (Ex px ) = 0

2mp (2mp + T )
mp (2mp + T )
m2x
p
=
=
p =
2(Ex px )
2(T + 2mp p)
T + 2mp (T + mp )2 m2p
mp
q
p =
T
1 T +2m
p

(12)
(13)
(14)

and plugging in for mp and T we get.


p =

0.94GeV
q
= 2.3GeV = E
1
1 1+2(0.94)

(15)

For the proton going in the opposite direction


E0 = p0 = Ex p = T + 2m p E = 1GeV + 2(0.49GeV ) 2.3GeV = 0.59GeV
(16)
(c) In this case the decay is symmetric so
p = p0 =

T
Ex
= E = E0 =
+ mp = 0.5GeV + 0.94GeV = 1.4GeV
2
2

(17)

Again using 4-momentum


(Ex , p~x ) (p0 , p~0 ) = (p , p
~ ).

(18)

Squaring then plugging in for p gives


m2x 2Ex p + 2px p cos = 0
cos =

Ex2 m2x
px
=
=v
px Ex
Ex

= cos1 (v) = cos1 (0.59) = 540

(19)
(20)
(21)

Paint gun

Astronauts Sally and Bob decide to settle one of their disagreements in a paint
duel. They fly their space shuttles on a collision course toward each other while
keeping their shuttles at constant speed. Each shuttle is equipped with paint
guns which are capable of emitting spray paint at high speed. Whoever gets the
most paint on their shuttle loses. A worker from from space station DS3 is there
to observe and referee the duel. Note: Express times in units of s (seconds),
velocities in units of c (speed of light), and distance in units of light seconds
= cs.
In the frame of the space station the following events take place:
event 1.) Sally fires her paint gun at x = 9cs and t = 2s.
event 2.) Bob fires his paint gun at x = 0cs and t = 0s.
event 3.) Bobs ship is hit by paint at x = 4cs and t = 5s.
event 4.) Sallys ship is hit by paint at x = 5cs and t = 6s.
NOTE: For this problem we let c = 1.
The 4-vectors for the events are as follows:
S1 = (2, 9, 0, 0)s

(22)

S2 = (0, 0, 0, 0)s

(23)

S3 = (5, 4, 0, 0)s

(24)

S4 = (6, 5, 0, 0)s

(25)

(a) Show that Sallys velocity in the rest frame of the space station is vS =
1/2c while Bobs velocity is vB = +4/5c.
(b) Find the intervals x0 and t0 between events 1 and 2 in Sallys rest frame.
(c) Determine if it is possible for events 1 and 2 to be causally connected.
(d) Determine the velocity of the paint from Bobs paint gun in Bobs rest
frame.
(e) Bob is trying to evade the rules of the paint duel, and sends a massage
on his communicator to a friends shuttle hidden behind Sallys shuttle.
Bobs communicator is using a frequency of f = 1010 Hz. Sallys receiver
is able to receive communications in the frequency range of 3 109 to
3 1010 Hz. Determine whether Sally is able to receive Bobs message.
Solution
(a) Using the two 4-vectors for Sally i.e. S1 and S4 we can determine Sallys velocity,
Vs , in the frame of DS3.
Vs =

(9 5)s
4
1
x
=
=
=
t
(2 6)s
8
2

Problem G4P7 from 2008Comps.

(26)

Using S2 and S3 we can determine Vb .


Vb =

(0 4)s
x
4
=
=
t
(0 5)s
5

(27)

(b) Using Lorentz transformations for space and time we get


x0 = (x t) x0 = (x t)
t0 = (t x) t0 = (t x)

Plugging in for S1 and S2 and using from part a.) Vs = 1/2 we get
x0
t0

1s
= q
1

1
4

1s
= q
1

1
4

(9 +

1
2
16
(2)) = 8s = s
2
3
3

(2 +

1
5
(9)) = s
2
3

(c) The easiest way to determine if two events are causally connected is to square the
interval S = (t0 , ~
x0 ). If the interval is timelike, t0 > x0 ; the two events
can be causally connected. If the interval is spacelike, x0 > t0 ; the pair of
events cannot be causally related. This is because events that have a spacelike
interval happen too far away in space and near in time so that an inertial frame
would have to travel faster than light in order to transport a clock between
events. Our interval is
16
5
(S)2 = (t0 )2 (x0 )2 = ( s)2 ( s)2
3
3

(28)

Seeing x0 > t0 ; we have a spacelike interval. Therefore, events 1 and 2 cannot


be causally connected.
(d) We know Bobs speed relative to DS3, and we can easily determine Bobs paints
speed using S1 = (0, 0, 0, 0)s (where the paint started) and S4 = (6, 5, 0, 0)s
(where the paint hit Sallys shuttle).
Vp =

(0 5)s
5
x
=
=
t
(0 6)s
6

(29)

And to determine the paints speed, (v 0 ), in Bobs frame we just use the definitions of velocity and Lorentz transformations as follows:
V0 =

dx

(dx dt)
dx0
Vp
dt
=
=
=
dx
dt0
(dt d)
1
Vp
1 dt

(30)

We are using c = 1 so = Vb , so plugging in we get:


V0 =

5
6

45
1
=
10
1 56 45

(31)

(e) The frequency Sally detects is


f=

1
1
=
,

t vt

(32)

(where v is the relative velocity between Sally and Bob) and rewriting the proper
frequency we get
1
1

f0 =
= =
(33)
0

t
Dividing equation (11) by equation (12):
r

f
t
1 v2
1+v
=
=
=
(34)
f0
(t vt)
1v
1v
so now all we need to do if find the relative velocity between Sally and Bob, and
we do this in the same way as in part d.)
v=

4
+1
Vb Vs
13
= 5 421 =
1 Vs Vb
14
1+ 52

Plugging this into equation 13 and solving for f :


s

27/14
f=
f0 = 27f0 = 3 3f0
1/14

(35)

(36)

f0 = fb = 1010 Hz so f = 3 3 1010 Hz. This means Sally cannot tell that Bob
is trying to cheat, because the frequency is just a bit too high for her receiver!!

Neutrino beams

(-Neutrino Beams) A beam of high energy -neutrinos is made at FermiLab


by forming a monoenergetic 2 GeV + beam, allowing the pions to decay by
the process + + + , and then deflecting the + particles out of the beam
with a strong magnet.
(a) Find the energy of neutrinos produced by this process in the rest frame of
the pions.
(b) Find the energy of a neutrino produced at an angle of 0 with respect to
the incident pion direction in the laboratory frame.
(c) Find the laboratory-frame emission angle (measured with respect to the
incident pion direction) at which the neutrino energy falls to half the
energy of neutrinos at = 0 , as calculated in part ii above.
Solution
(a) Four-momentum conservation in the CM frame
(m 0)
(m 0)

(Ecm

(Ecm p
~) + (Ecm p
~)
(Ecm
m2

p
~)

m2 2m Ecm

Ecm

(37)

p
~)

m2 m2
(134.98 M eV )2 (105.66 M eV )2
=
= 26.14 M(38)
eV.
2m
2 134.98 M eV

(b) Four-momentum conservation in the lab frame


(E p
~ )

(E p
~ ) + (E p
~ )

(E p
~ ) (E p
~ )

(E p
~ )

m2 2E E + 2p E cos ()

m2

m2 m2
m2 m2

=
2(E p cos (0 ))
2(E E2 m2 )

(134.98 M eV )2 (105.66 M eV )2
p
= 774 M eV.
2(2000 M eV (2000 M eV )2 (134.98 M eV )2 )

(c) Solving equation 40 for and using E2 for the new value of the neutrino energy
we get
 2

m m2 + E E
2
= cos1
E E m2
!
(105.66 M eV )2 (134.98 M eV )2 + 2000 M eV 774 M eV
1
p
= 3.87 .
= cos
774 M eV (2000 M eV )2 (134.98 M eV )2

Problem G4P18 from 2008Comps.

(39)
(40)

Star around cluster

How does the orbital velocity ~v of a bright star that is in a circular orbit
around a cluster of galaxies depend on the orbit radius ~r and the system mass
M , under the following assumptions?
(a) Under the assumption that the total attractive mass MV is in the form of
~ V < ~r.
visible stars that are distributed symmetrically within a radius R
(b) Under the assumption that the mass of visible stars is negligible and that
the orbit occurs within a uniform sphere of dark matter of mass MD with
a radius RD > ~r.
(c) It is observed that for a number of bright stars orbiting at various distances
around a galactic cluster, v(r) r and that v 2 r >> GMV . Discuss these
results in the context of evidence for dark matter.
Solution
(a) To do this part all we have to do is balance the force due to gravity and the
centripetal force due to the stars orbit.
M v2
GMv M
=
2
r
r
and solve for v

r
v=

GMv
r

(41)

(42)

(b) For this part we assume that the only mass the affects the orbit is that which is
within the orbit radius, and we know it is a uniform sphere so:
(r) = (RD ).

(43)

Now just using the the fact that = M/V , and letting the inner mass be MDr
we get:
MDr
MD
MD r 3
=
MDr =
(44)
3
Vr
VD
RD
Plugging this into our forces equation gives us
GMDr M
GMD M r3
M v2
=
=
3
r2
r 2 RD
r
and solve for v

s
v=r

GMD
3
RD

(45)

(46)

(c) For this part we add the forces from the visible stars and the dark matter like so
GMv M
GMD M r3
M v2
=
+
3
r2
r 2 RD
r
Problem G5P12 from 2008Comps.

(47)

rearranging and multiplying by r


v 2 r = GMv +

r3
GMD
3
RD

(48)

So if Mv is negligible compared to MD (and the orbit is not close to the center


i.e. r  RD is not true) then yes, v 2 r >> GMV . v(r) r can be seen from
equation (6). So if this is how observed stars act then it does indeed point to
dark matter.

Superluminal objects

Studies of the quasar 3C345 using long-baseline radio telescopes have shown
that the object emitted a radio-bright fireball that, in observations made
between 1969 and 1976 separated from the main body of the quasar and moved
an apparent distance of 56 light years during the observations. Taken at face
value, the hot object would be traveling at 8 times the speed of light. Assume
that the quasar is a distance D from the Earth and that the hot object is moving
away from the quasar in the general direction of the Earth at a relativistic
velocity c, with its direction of travel making a small angle with the line of
sight between the quasar and the Earth.
~ between the quasar and fireball
(a) Calculate the actual separation distance R
and the apparent separation distance x from the quasar, as seen by an
observer on Earth, as the hot object moves away from the quasar in the
~ perpendicular to the line
time interval t. Here x is the component of R
~
of sight, and t, R and x are in the reference frame of an observer on
Earth.
(b) Calculate the transit time T1 to the Earth of radio waves emitted when
the object is ejected from the quasar, the transit time T2 of radio waves
emitted by the hot object a time t later, and the time difference T
between the arrival at Earth of the waves from the two events.
(c) Assuming that is a small angle, use appropriate approximations to estimate the apparent velocity ~v of the object. Explain how it can exceed the
velocity of light.
Solution
(a) Let c = 1. Then R = t, and x = t sin .
(b) If the distance is D then the time it takes the radio waves to get to earth is
T1 = D/c = D, and for T2 we need to take into account how for the fireball has
moved towards us in the time, t that has elapsed. So, T2 = D t cos , then
T = D D + betat cos = betat cos .
(c) The apparent velocity can be found by comparing distances. The apparent distance is the apparent velocity times the time it takes the light to get there,
Va teye , and this is equal to the real time times the real velocity.
Va teye = t

(49)

or plugging in for t = D and teye = D t cos .


Va (D D cos ) = D

(50)

solving for Va
Va =
Problem G5P12-2 from 2008Comps.

1 cos

(51)

Which for the small angle approximation is just


Va =
So if is small and is more than
then the speed of light.

1
2

.
1

(52)

then the apparent velocity is in fact larger

Two protons

Two protons approach each other head on at speeds of 0.5c relative to the
reference frame S.

(a) Find the speed of one proton in the rest frame S 0 of the other. Recall that
the relativistic addition rule for velocities in, say, the x
-direction is given
by:
ux v
(53)
u0x =
1 ux v/c2
where v is the relative velocities of S and S 0 .
(b) What is the kinetic energy of the two protons in the frame S.
(c) What is the kinetic energy of the two protons in the frame S 0 .
(d) What is the momentum of the moving proton in the frame S 0 . The mass
of the proton is 938.28 MeV.
Solution
(a)
ux =

(0.5c) (0.5c)
1+

(0.5c)(0.5c)
c2

= 0.8c

(54)

(b) We know E = KE + m and that E = m, so KE = m m = ( 1)m. Also


let c = 1.
!
1
KE = (1)m = p
1 m = (0.1547)(938.34M eV ) = 145.16M eV
1 (0.5)2
(55)
(c) If we are in the S 0 frame then of course the KE of one of the protons is zero, and
to find the KE of the other we just use the same formula as we used in part b)
and the relative velocity found in part a).
!
1
1 m = (0.667)(938.34M eV ) = 625.87M eV
KE = ( 1)m = p
1 (0.8)2
(56)
(d)
p = mv = (1.667)(983.34M eV )(0.8) = 1251.12M eV

Problem G5P17 from 2008Comps.

(57)

K mesons
K + mesons can be photoproduced by the reaction
+ p K+ +

(a) Determine the threshold (minimum) photon energy, as measured in the


laboratory (the rest-frame of the proton),for the above reaction to occur.
(b) Determine if it is possible for either
(i) the K + or
(ii) the to be at rest in the lab frame, and determine for what
photon energy this could happen.
Solution
(a) Using the invariant mass of the initial system in the lab frame we have
M 2 = m2p + 2Emp

(58)

After the colission we get the invariant mass in the center of mass. Since we want
the threshold energy the particles are at rest in the center of mass, therefore
M 2 = (mK + + m )2

(59)

from equation 1 and 2 we get


E =

(mK + + m )2 m2p
2mp

(60)

Pluging the values in this equations we get that


E = 911M eV
(b)
(i) First we write the four vectors for the reaction in the lab frame:
(E , E ) + (mp , 0) = (mK + , 0) + (E , p )

(61)

To pretty up the math, we move the K + term to the other side before squaring.
[(E , E ) + (mp , 0) (mK + , 0)]2 = (E , p )2

(62)

mp 2 + mK + 2 + 2E mp 2E mK + 2mp mk = m2

(63)

And solving for E :


E =

mp mK +
m2

2(mp mK + )
2

Problem modern1 from 2008Comps.

(64)

Plugging in the rest-mass energies (in

M eV
c2

), We get

E = 1178.05M eV
This energy is a positive number, and above the threshold energy. Therefore,
producing a kaon at rest is possible.
(ii) Part (ii) follows the same as in part (i) except that our mK + and our
m are interchanged. Therefore we can immediately write down the solution
for E .
m2K +
mp m
E =

(65)
2(mp m )
2
Which gives an energy of
E = 600.87M eV
This energy is negative, hence it is not possible to produce a stationary lambda
particle in the rest frame of the proton.

Muons

A relativistic beam of monochromatic muons produced in the upper atmosphere is incident vertically on the earths surface at velocity v c, where c is
the speed of light.
(a) Find the ratio of muon number reaching the ground to the number at a
height H above sea level. Assume that the beam moves at a constant
velocity and that the attenuation of the beam is due entirely to the spontaneous decay of the muons. Express your answer in terms of the energy
E of a muon as measured by an observer at rest on the ground, the muon
rest- mass m , and the muon rest-frame lifetime(i.e., decay time) 0 .
(b) Explain in one sentence why the ratio must be the same for an observer
comoving with the beam. Re-express your answer to (1) entirely in terms
of parameters measured by a comoving observer.
Solution
(a) The decay of muons dN at any time is directly proportional to the number of
muons present at that moment N and to the differential time dt (where we
consider N to be constant over the interval dt). Thus,
dN N dt
dN is negative as dt increases, requiring a negative sign on the right hand side.
The proportionality constant is the probabilty for a muon to decay per unit time
interval, hence it is 1/ , where is the mean lifetime of the particle.
1
dN = N dt

Doing the usual integration, we get


N = N0 exp(t/ )
where N0 is the initial number of muons.
For a person on the earth, the rest lifetime of the muon 0 is time dilated by a
factor of . Hence,
= 0
The person on the earth measures the muons velocity vE as the ratio of the
distance travelled H to the time taken to travel the same, tE , hence
vE =

H
tE

Hence, we the ratio as


N
= exp(H/vE 0 ) = exp(Hm c2 /vE 0 E )
N0
where m is the rest-mass of the muon and E is the energy of the muon
measured by a person on the earth.
Problem ModernPhysics1 from 2008Comps.

(b) How can the ratio depend on the frame of the observer? The time intervals in
the frame of the earth and that of the muon are related byt = tE
Hence,
H
v
H
=
=
tE
t

where v is the self-velocity measured by a person co-moving with the beam.


We substitute for vE in terms of v in the ratio derived in P art(1). Hence,
vE =

N
= exp(Hm c2 /vE 0 E ) = exp(H/v 0 ) = exp(t /0 )
N0
where t is the time interval taken by the muons to fall through a height H as
measured by a person co-moving with the muons.

10

Doppler effect

The radar speed detector operates on a frequency of 109 Hz. What is the beat
frequency between the transmitted signal and the one received after reflection
from a car moving at 30m/sec?
Solution
Suppose the car is moving towards the radar with velocity v. Let the radar frequency
be 0 and the frequency of the signal as received by the car be 1 . The situation is
the same as if the car were stationary and the radar moved toward it with velocity v.
Hence the relativistic Doppler effect gives
s

1 + v/c
v
1 = 0
' 0 1 +
1 v/c
c
correct to the first power of v/c. Now the car acts like a source of frequency 1 , so
the frequency of the reflected signal as received by the radar (also correct to the first
power of v/c) is
s




1 + v/c
2v
v
v 2
' 1 1 +
' 0 1 +
' 0 1 +
2 = 1
1 v/c
c
c
c
Thus the beat frequency is
2 0 = 0

2 30
2v
= 109
= 200 Hz.
c
3 108

Short questions 2

(a) Estimate the collision frequency per molecule (in sec1 ) for the gas in this
room.
(b) Demonstrate the validity of the dipole approximation for typical atomic
transitions; do this by computing the characteristic difference in the photon phase over atomic dimensions.
(c) Photons and neutrinos (energy 10 MeV) are emitted simultaneously from
a supernova at a distance of 1.5 1025 cm. If the neutrinos arrive here 100
sec after the photons, what is their mass?
(d) Neutral atoms interact by electrical forces. Explain qualitatively how this
happens and derive the form of the interaction at large separation.
Solution
(a) The mean free path of the molecules is
1

2d2 n
where d is the diameter of the molecule and n is the number density of the gas.
The mean speed is
r
8kT
v =
,
m
so the frequency is
r

8kT
p
v
pd2
= 16
.
f= =
2d2

m
kT
mkT
Since p 106 Pa, d 1
A = 1010 m, k = 1.38 1023 J/K, T 300K, m
28103 Kg
,
61023
f 5 108 /sec.
(b) Incoming EM wave (photons):
E = E0 ei(krt)
Since the dimension of atoms is r 1
A, and 103
A 104
A, then
kr =

2
r 103  1,

eikr 1

which is dipole approximation.


Problem 6 from UCLA 1987Fall.

(c) The distance is


D = ct1 = vt1 + vt,
so
v = c v =

c2 t
= 2 1013 c.
D

The momentum of the neutrino is


p = mv =
then
m=
(d) The dipole field E

E
E
=
vc
vc
1
,
r3

r
1

E
,
c

E
v2
2
2cv = 6.3eV /c2 .
2
c
c v

and the induced dipole p


U pE

1
.
r6

1
,
r3

thus the interaction

Numerical values 2
Give approximate numerical values for the following (be sure to give units):

(a) Escape velocity from the earth


(b) Ground state energy of muonium (+ , atom)
(c)

P (1)n1
n

(d) Spin of the deuteron


(e) Lifetime of the neutron
(f) Specific heat of Cu at room temperature (assumed greater than Debye
temperature)
(g) The orbital period of a planet (Saturn) which is 3 times as far from the
sun as the earth is.
Solution
(a)
r

GM
7.9km/s
Re

(b)

where
=

e4
2
h2

105M eV /c2
m+ m
m

m+ + m
2
2

so
13.6eV

105/2
1428eV.
0.51

(c)

X
(1)n1
= ln 2
n
n=1

(d) Spin of deuteron is S = 1.


(e) Life time of neutron is 1000s.
(f) 3R 6cal/(mol*K)
(g) Since R3 /T 2 =const, so the period of Saturn is

3
1year ( )3/2 = 27year 5.2years.
1

Problem 2 from UCLA 1989Spring.

Neutron density flow


The diffusion equation

U
= 2 U + U
t
provides a crude model to describe the flow of neutron density U (t, x, y, z) in a
block of uranium. Here, is the diffusion constant, and governs the rate at
which free neutrons are created through collision.
(a) Use separation of variables to obtain a solution in a cube of size D: 0 <
x, y, z < D, given an arbitrary distribution U (0, x, y, z) = (x, y, z) and
the boundary condition that U 0 on the surface of the cube for all t 0.
(b) Use the solution of part (a) to determine the critical size, Dc , of such a
cube such that U will grow exponentially with time if D > Dc .
Solution
(a) Let U = X(x)Y (y)Z(z)T (t), then
X 00
Y 00
Z 00
T
/T = (
+
+
) + = 2
t
X
Y
Z
2

T e

also

X 00
Y 00
Z 00
2
+
+
=
X
Y
Z

Let Z sin kx x, Y sin ky y, Z sin kz z,


kx2 + ky2 + kz2 =

And U (t, x, y, z) = 0 at boundary,

kx = l
D

ky = m
D

kz = n
D
hence
(l2 + m2 + n2 )
or

2
2
=
D2

2 2
(l + m2 + n2 )
D2
X
lx
my
nz [ 22 (l2 +m2 +n2 )]t
D
U (t, x, y, z) =
Almn sin
sin
sin
e
D
D
D
2 =

l,m,n

with U (0, x, y, z) = (x, y, z),


Z Z Z
23
lx
my
nz
Almn =
(x, y, z) sin
sin
sin
dxdydz
D3
D
D
D
Z D
Z D Z D
8
lx
my
nz
=
dx
dy
dz(x, y, z) sin
sin
sin
D3 0
D
D
D
0
0
Problem 19 from UCLA 1990Spring.

(1)

l, m, n = 1, 2, 3, ...
(b) For exponentially grow,
2 2
(l + m2 + n2 ) > 0
D2
3 2
>
D2
3 2
D2 >
= Dc2

r
3
Dc =
.

Model for a star

Consider as a hypothetical model for a star an ideal gas sphere in hydrostatic


equilibrium at a uniform temperature T0 . Assume spherical symmetry.
(a) Find an equation for the pressure derivative dP/dr at radius r, in terms
of the gravitational constant G, the density (r) at radius r and the mass
interior to r, M (r).
(b) Assume the star undergoes a uniform contraction: the distance between
any two mass elements changes by the same fraction y. Find the change
in the stars temperature necessary to maintain hydrostatic equilibrium.
You may neglect the effect of radiation pressure.
(c) Assuming the surface of the star emits radiation as a black body, and based
on your answer in (b): How will the color of this change as it contracts?
How will the luminosity L change? (L =energy radiated per unit time).
Solution
(a) Equation for equilibrium
(P (r dr) P (r))4r2 = Gdm

M (r)
r2

dm = dV = 4r2 dr
GM (r)
dP

=
dr
r2
0
(b) Change scale of the length r r/y = r , (y > 1), so y 3 = 0 , M M ,

dP 0
= G0 M (r0 )/r02
dr0

dP 0 = Gy 3

M dr
GM
=
dry 4
r2 /y 2 y
r2

dP 0 = y 4 dP, P 0 = y 4 P
Since P = nkT for ideal gas, P 0 = n0 kT 0 , n0 = y 3 n,
T 0 = yT T 0 = yT0 T = (y 1)T0
(c) When the temperature of the star changes, the radiation spectrum shifts. Now
y > 1, the star contracts, the temperature increases, from the formula
hmax = 2.82kT,
we know max will increase, so the color of this star changes to higher frequency
light like blue.
L = T 4 4R2 = 4R2 T 4
T 04 R02
y4
L0
= 4 2 = 2 = y2 ,
L
T R
y
the luminosity increases.
Problem 4 from UCLA 1990Fall.

Differential equation

(a) J(z) is a solution of the differential equation zJ 00 + J 0 = zJ(z) = 0 with


J(0) = 1. Give the first two non-zero terms in the series expansion of J(z)
about z = 0.
(b) Find the first two terms in the asymptotic expansion of
Z 2
exp{t cos }2 J()d
F (t) =
0

as t +.
Solution
(a)
zJ 00 + J 0 + zJ(z) = 0
J(0) = 1
Let
J(z) = 1 + az + bz 2 + cz 3 + dz 4 + ...
Put this into the equation, then
12dz 3 + 6cz 2 + 2bz + a + 2bz + 3cz 2 + 4dz 3 + z + az 2 + bz 3 + ... = 0
1
1
a = 0, b = , c = 0, d =
4
64
1
1 4
J(z) = 1 z 2 +
z + ...
4
64
(b)

Z
F (t) =

exp{t cos }2 J()d

when t ,

Z
F (t)

exp{t(1

F (t)

et

2
1
)}2 (1 2 )d
2
4

exp(

The first term is


F (t)1

=
=
=
=

Problem 9 from UCLA 1990Fall.

et
p
( t/2)3

et
p
( t/2)3

et

t2 2
1
) (1 2 )d
2
4
t
2

exp(x2 )x2 dx

1 3
23/2 ( ( ))
2 2
r
et
2 t3/2
t3/2

exp(02 )02 d0

The second term is


F (t)2

et

Z
0

et
p
4( t/2)5
t

=
=
=

t2 2 1 2
) d
2
4

Z t
2
2
exp(02 )04 d0

exp(

e
p
4( t/2)5

exp(x2 )x4 dx
0

et 5/2 1 5
2 ( ( ))
2 2
4t5/2

3 2 et

8 t5/2

So the first two terms are


r
F (t) =

et
3 2 et

2 t3/2
8 t5/2

Asymptotic expansion
Consider the integral expression
4a
J(a) =

dx
0

xex
1 ea/x

(a) What is the limiting asymptotic form of J(a) as a 0? Keep at least two
terms in the expansion.
(b) What is the limiting asymptotic form of J(a) as a ? Keep at least
two terms in the expansion.
Solution
(a) We use a 0,
a
a2
a3
2 + 3 + ...)1
x
2x
6x
a
a2
x
+ 2 + ...]1
= [1
a
2x
6x
x
a
= [1 +
+ ...]
a
2x

(1 ea/x )1 = (

We have then
Z
4
a 2 x2
J(a)
dx(1 +
)x e
2x
0
Z
Z
2
4
a x2
[
x2 ex dx +
xe
dx]
2 0
0

a1
4
+
]
[
22
4
a
1+

(b)
a , ea/x  1
(1 ea/x )1 1 + ea/x
Hence

Z
Z
2
2
4a
J(a) [
xex dx +
dxxe(x +a/x) ]
0
0
We use saddle point in the second integral,
f (x) = x2 +

a
x

a
a
x0 = ( )1/3
x2
2
2a
00
f (x) = 2 + 3
x

f 0 (x) = 2x

Problem 8 from UCLA 1992Fall.

a
3
f (x0 ) = ( )2/3 + 21/3 a2/3 = 1/3 a2/3
2
4
f 00 (x0 ) = 6
We have then
Z

dxxe(x

+a/x)

dxx0 ef (x0 ) ef

(x0 )

(xx0 )2
2

x0 ef (x0 ) (
So

00

2 1/2
)
f 00 (x0 )

4a 1
a

3
J(a) [ + ( )1/3 ( )1/2 exp( 1/3 a2/3 )]
2
3
4
2

Model of the sun

In the following problem order of magnitude estimates will suffice. Model the
sun as a uniform constant density ( = 1.4 g cm3 ) sphere of ionized hydrogen
with radius R 7 1010 cm. For the purpose of estimating the internal energy
of the sun, assume a characteristic temperature of T 4.5 106 K. (Note that
this is not the surface temperature of the sun.) Further assume that local
thermodynamic equilibrium between the radiation field and the matter is a
good approximation. The typical photon scattering cross section can be taken
to be roughly the Thomson value 1024 cm2 .
(a) How long would it take a photon to random walk from the center to the
surface of the sun in this model?
(b) Estimate the radiant power emitted by the sun, assuming that all energy
escapes through the process of photons random walking from the center
of the sun.
(c) Compare the kinetic energy density of the gas to the energy density of
radiation. Find the time taken for the sun to radiate away its internal
energy.
(d) How long could the sun shine with the luminosity estimated in part (b) if
it were powered by hydrogen burning? In this process 4 protons combine
to form 4 He through a series of strong, weak, and electromagnetic interactions. The binding energy of 4 He relative to 4 free protons is roughly
27.6 MeV. Assume that, over its lifetime, of order ten percent of the suns
mass is available for hydrogen burning.
Some useful constants:
Radiation density constant a = 7.6 1015 erg cm3 K4
Boltzmann constant kB = 1.4 1016 erg K1 .
Solution
(a) For 1-dimensional random walk the mean square displacement after N scatterings
is given by
hx2 i = N 2
where N = number of scatterings, and is the mean free path.
=

1
1
=
1cm
n
1.4 6.02 1023 1024

Now in 3D, only 13 of the scatterings contribute to the mean square displacement
in a given direction; so radial mean square displacement is
r2 =
Problem 1 from UCLA 1993Spring.

N 2

so number of scatterings required for escape is Nes =


between scatterings is /c, so escape time is
tes = Nes

3R2
,
2

and mean time

3R2
=
5 1011 s 2 104 yrs.
c
c

(b) Radiation energy density ur = aT 4 31012 erg cm3 , so total radiation energy
in the sun is
4
Ur = ( R3 )aT 4 4.5 1045 ergs.
3
Estimate power by assuming Ur is released on time scale tes
P

Ur
1034 erg s1
tes

(c) The proton density np = NA = 8.4 1023 cm3 . Thus, the electron plus
proton density is n = 1.7 1024 cm3 , and the total thermal kinetic energy is
approximately
4
3
Uk = R3 ( nkT ) 2.3 1048 ergs
3
2
or Uk 500Ur . So that it must take about 500 times larger than tes to radiate
away thermal kinetic energy or 3 1014 s 107 yrs, the Kelvin-Helmholz
time

Model of a star

Model a star as an ideal gas in hydrostatic equilibrium, with uniform temperature = kT. Assume spherical symmetry. (Consider only hydrostatic pressure
and neglect other sources, like e.g. radiation pressure.)
(a) Based on equilibrium for a shell, find an equation for the density derivative
d/dr at radius r, in terms of the gravitational constant G, the density (r)
at radius r, mass interior to r, M (r), temperature , and gas molecular
mass . (The relationship between density and number density is = n.)
(b) Assume the star undergoes a uniform contraction: the distance between
any two mass elements changes by the same fraction y < 1, so that for
every radius r1 = yr. Find the stars new temperature 1 necessary to
maintain hydrostatic equilibrium. Assume the whole surface area of the
star emits radiation as a black body.
(c) How will the color of this star change as it contracts?
(d) How will the luminosity L change as it contracts? (Luminosity is the total
energy radiated per unit time).
Solution
(a) Consider the pressure and gravitational forces on a shell at radius r:
P A = (P (r) P (r + dr)) 4r2 = Fg =

GM (r)(r)4r2 dr
GM (r)dm
=
2
r
r2

so that

GM (r)(r)
dP
=
dr
r2
Substituting the ideal gas law P = n = /, we find
d
GM (r)(r)
=
dr

r2
(b) Rescaling the quantities as r1 = yr, we find 1 = /y 3 and M1 (r1 ) = M (r). Thus
the new equation of hydrostatic equilibrium is
d1
GM1 1
=
.
dr1
1 r12
Tabulating powers of y we find
d 4
GM 5
y =
y .
dr
1 r2
Thus for consistency we must have
1 = /y
and the temperature rises as the star contracts.
from Kevin M. Huffenberger

(c) The location of the peak of the Planck spectrum is given by Wiens displacement
law,
max T = constant = 2.898 103 m K,
so that the wavelength of peak emission shrinks as the star contracts and the
temperature rises, as max y. This shifts the color toward the blue end of the
spectrum.
(d) The Stefan-Boltzmann law j = T 4 gives the energy per time radiated per unit
area. So the total luminosity is
L = jA = T 4 4R2
where the radius R provides the surface area of the star. Thus,
L1
4 R2
= 14 12 = y 4 y 2 = y 2 .
L
R

van der Waals gas 2


Consider a real gas described by the van der Waals equation of state

a
P + 2 (v b) = RT
v

where P is the pressure, T is the temperature, v is the volume per mole of the
gas, R is the universal gas constant, and a and b are constants.
(a) Briefly explain the physical origin and meaning of the constants a and b.
(b) The coefficient of thermal volume expansion is defined as = v 1 (v/T )P .
Show that for the van der Waals gas
=

Rv 2 (v b)
.
RT v 3 2a(v b)2

(c) Show that the isothermal compressibility, T = v 1 (v/P )T , for the


van der Waals gas is given by
T =

v 2 (v b)2
.
RT v 3 2a(v b)2

(d) Obtain (P/T )v and verify that



 
 

P
T
v
= 1.
T v v P P T
Solution
(a) First examine the constant b. In the equation the term v b takes the place of
the volume per mole v in the usual ideal gas law P v = RT . This is because the
gas molecules are not pointlike but have a finite size, and so we can interpret b
as the volume taken up by a mole of these molecules if they are not moving.
The term involving a actually corrects the ideal gas law, which assumes that the
molecules do not interact with each other, and comes from these interactions in
a real gas. Since they go down like the volume per mole squared, they actually
go like 1/r6 where r is the separation of the molecules, and so are of the van
der Waals type. Note that the pressure is reduced by the term proportional to
a, so each molecule is in fact attracted by the other molecules in the gas which
results in no net force except near the walls. Near the walls the molecules feel
a net force away from the wall and so the pressure is reduced.
(b) This is an exercise in finding total differentials. If we are keeping P fixed then
the total differential of the gas law takes the form

a
2a
R dT = 3 dv (v b) + P + 2 dv.
v
v
Problem OpaTh-F99-3 from Courses-SC.

Note that the expression we are to find does not involve P , so eliminate P +a/v 2
using the gas law
R dT =


2a
RT
dv
dv (v b) +
dv = 3
RT v 3 2a[v b]2 ,
v3
vb
v (v b)

so that
=

1
v

v
T

v 2 (v b)
.
2a(v b)2


=
P

RT v 3

(c) We now do the same again this time keeping T constant and letting P vary, so
that

a
2a
0 = dP (v b) 3 dv (v b) + P + 2 dv,
v
v
and using the same substitution we find
dP (v b) =
so that
T =

1
v


dv
RT v 3 2a[v b]2 ,
b)

v 3 (v

v
P


=
T

v 2 (v b)
.
RT v 3 2a(v b)2

(d) If v is constant then the above collapses to


dP (v b) = R dT,
so that

P
T


=
v

R
,
vb

and


P
T

 
v

T
v

 
P

v
P


=
T

R T
R 1
(vT ) =
= 1.
v b v
vb

Convex lens and concave mirror

An object is 15 cm to the left of a thin convex lens of focal length 10 cm. A


concave mirror of radius 10 cm is 25 cm to the right of the lens.
(a) Find the position of the final image formed by the mirror and the lens.
(b) Is the image real or virtual, upright or inverted?
(c) Show on a diagram where your eye must be to see this image.
Solution
(a) Careful use the lens equation, which also works for spherical mirrors, and that for
the magnification of these optical systems will yield the results we need. The
lens equation
1
1
1
+ 0 =
s
s
f
works for lenses and spherical mirrors, where s is the distance of the object from
the lens/mirror, s0 is the distance of the image from the lens/mirror, and f is
the focal length which is the image distance when s = . These quantities are
taken to be positive when the object, image, or center of curvature lies on the
real side of the optical element. For lenses the real side is the incident side for
objects and the transmission side for images and centers of curvature. Positive
s0 means a real image, negative s0 means a virtual image. The magnification is
m = s0 /s, and if it is negative this means the image is inverted.
Applying this to the lens we have
1
1
1
+ 0 =
,
15 cm
s1
10 cm
so that s01 = 30 cm. Note that since s0 is positive this image is real and inverted.
Now this image would form 5 cm beyond the surface of the mirror, so it is not
on the real side of the mirror. Note that for a spherical mirror the focal length
is half the radius, so the equation for the mirror looks like
1
2
1
+ 0 =
,
5 cm
s2
10 cm
so that s02 = 2.5 cm. Since this is positive we know that it occurs on the real side
of the mirror inside the focal length, 2.5 cm to the left of the mirror, and so is a
real image. Since for the mirror s02 is positive and s is negative the magnification
is positive so the image remains inverted with respect to the object.
This real image will now behave like an object to the lens, so applying this
equation once more we find
1
1
1
+ 0 =
,
22.5 cm
s3
10 cm
so that s03 = 18 cm.
Problem OpaTh-F99-4 from Courses-SC.

(b) This image is on the transmission side of the lens and so is a real image, and is
inverted relative to the image from the mirror. The net result is an upright real
image 18 cm to the left of the lens.
(c) Since light rays appear to emanate from this real image but are moving away
from the lens, we will need to place our eye to the left of the real image to see
it.

Camera

A 35 mm camera has a negative size of 24 mm by 36 mm. It is to be used


to take a picture of a person 175 cm tall in which the image of the person just
fills the height (24 mm) of the film. How far should the person stand from the
camera if the focal length of the lens is 50 mm?
Solution
The lens equation gives
1
1
1
+ 0 = ,
s
s
f
and the magnification is m = s0 /s. Using the desired magnification of m = 2.4/175
(note it will be negative for a converging lens) we can solve these two equations for s,
1
1
+
s
ms


1 m + 1
s
m

=
=

1
f
1
f


m + 1
m

Problem OpaTh-F99-5 from Courses-SC.


= 5 cm

2.4/175 + 1
2.4/175


= 369.6 cm.

Cross section

Consider scattering in the central potential (r = |~r|)


u 
h  r i
0
U=
exp
r
a
(1) Calculate the differential cross section in the first Born approximation
2
Z

m2
d
3
d rU exp (i~q ~r)
=
4

2
d
4 h
where ~q = ~kout ~kint is the difference of the wave vector of the incoming
and outgoing particle.
(2) Find the total cross section tot (k). Hint: q = |~q| is related to the scattering
angle by q = 2k sin 2
Solution
(1) We use spherical coordinates with the z-axis in ~
q direction. The d integration
gives 2. So, with x = cos (this is not the scattering angle)
Z

h r
i
d3 r
exp
i~
q ~r
r
a

=
=
=
=

h r
i
dx exp
(iqr) x
a
0
+1
Z
i
h r
io
h r
2i n
+ iqr exp
iqr
dr exp
q 0
a
a
Z
2i
dr {exp [c r] exp [c+ r]}
q 0


1
2i 1

q
c+
c

rdr

where c = a1 iq = a/ (1 + iaq). Now,




2i 1
1
4a2

=
.
q
c+
c
1 + a2 q 2
Therefore,
Z
h r
i 2
m2 u20 d3 r
m2 u20 a4
=
exp

i~
q

~
r

r
a
4 2
h4
4 (1 + a2 q 2 )2
h
(2) As d = dd cos , the d integration is trivial (= 2) and the relevant integral is
(with y = sin2 (/2) and using cos () = cos2 (/2)sin2 (/2) = 12 cos2 (/2))


+1
1

d
d

d cos


1 + 4k2 a2 sin2 (/2)

2

2
0

=
=

1

1
dy

=

2
2 a2 (1 + 4k 2 a2 y)
2
2
2k
[1 + 4k a y]
0


1 + 1 + 4k2 a2
1
1
+
=
2k2 a2 (1 + 4k2 a2 )
2k2 a2
2k2 a2 (1 + 4k2 a2 )
2
1 + 4k2 a2

The total cross section is then


Z
=

d =

4m2 u20 a4

h2

d
16m2 u20 a4

2 = 4
2
2
2
(1 + 4k2 a2 )
h
1 + 4k a sin (/2)

Ideal gas in a cycle


An ideal gas ( = 1.5) follows the cycle shown in the figure.

(a) If P1 = 1 atm, V1 = 100 L, and T1 = 20 C, find the amount of gas


considered at point 1.
(b) The gas is then heated at constant volume to point 2, where the pressure
is P2 = 2 atm. What is the temperature of the gas there?
(c) Next, the gas is isothermally expanded to point 3, such that P3 = P1 = 1
atm. What is the work W23 done during the isothermal expansion?
(d) How is the result modified if an adiabatic rather than isothermal expansion
is considered between points 2 and 3?

Solution
(a) Use the ideal gas law and the value in the appropriate units R = 0.0821 L atm/(mol K),
and using T1 = 293 K, we have
n=

P1 V1
= 4.16 mol.
RT1

(b) Using the ideal gas law again, we have that since V is constant
P1
T1

T2

P2
T2
P2
T1 = 586 K.
P1

(c) Lets first find V3 . Since the temperature is unchanged from T2 , the volume is
given by the ideal gas law
P3 V3 = P2 V2 = P2 V1
Problem OpaTh-F99-6 from Courses-SC.

to be

P2
P2
V1 =
V1 = 2V1 = 200 L.
P3
P1
For an isothermal expansion we have
Z V3
P dV
W23 =
V3 =

V1

and using the ideal gas law we have that P = nRT2 /V , so that
Z V3
1
W23 = nRT2
dV = nRT2 ln(V3 /V1 )
V1 V
=

(4.16 mol) (0.0821 L atm/(mol K)) (586 K) ln(2) = 139 L atm.

(d) For an adiabatic expansion we have no heat added to the system, and the expansion follows
P V = constant,
where = Cp /Cv = 1.5. The volume is now changed to
V30


=

P2
P3

1

= 22/3 V1 .

The work integral is modified to


W23

P2 V2

V30

3/2

3/2

dV = P2 V1

2V 1/2

iV30

V
V1




1/2
1/2
1/3
V1
2
V1
= 2P2 V1 1 21/3


2(2 atm) (100 L) 1 21/3 = 82.5 L atm.
V1

3/2
2P2 V1

Two lenses

Two lenses are separated by 35 cm. An object is placed 20 cm to the left of


the first lens, which is a converging lens of focal length 10 cm. The second lens
is a diverging lens of focal length 15 cm. What is the position of the final
image? Is the image real or virtual? Upright or inverted? What is the overall
magnification of the image?

f2

f1
s1

s2

s1

s2

Solution
We can find everything we need from the lens formula(e)
1
1
1
s0
+ 0 = , m= ,
s
s
f
s
being careful to use a negative f for the diverging lens (since the focal point of light
on the incident side is on the incident side and not the transmitted side, it is therefore
negative), and use the figure to check our answers. For the first lens we have
1
1
1
1
1
1
=

=
,
s01
f1
s1
10 cm
20 cm
20 cm
so that the first real image is 20 cm from the lens and has
m1 =

s01
= 1,
s1

so that the image is the same size as the object and inverted.
For the second lens we have
1
1
1
1
1
2
=

=
,
s02
f2
s2
15 cm
15 cm
15 cm
so that the image is virtual (it is on the incident side, since s02 < 0) and 7.5 cm to the
left of the second lens. The magnification of the second lens is
m2 =

s02
7.5 cm
1
=
= ,
s2
15 cm
2

Problem OpaTh-F99-7 from Courses-SC.

so that the combined magnification is m = m1 m2 = 0.5 and the virtual image is


inverted with an overall magnification of 0.5.

Cylinder with piston

The cylinder shown in the figure has a piston of mass M that can slide without
friction. The area of the piston is S and the cylinder is filled with an ideal gas
( = 1.5), with an initial volume V , and an initial pressure P . Assume that the
outside pressure on the piston is zero (vacuum).

(a) Determine the initial acceleration of the piston.


(b) Calculate the velocity of the piston after it has moved a distance L, assuming that the gas is thermally isolated (adiabatic).
(c) What distance would the piston have to move for the temperature of the
gas to drop to one half of the original value?
Solution
(a) The initial force on the piston is given by the initial pressure P times the area
of the piston to which this pressure is applied, F = P A. Then the initial
acceleration is
PA
F
=
.
a=
M
M
(b) Adiabatic expansions follow the rule P V = constant, where is the ratio
CP /CV . We can find the constant involved by using the initial condition, so
that at some later time when the pressure is P 0 and the volume V 0 we have
P 0 V 0 = P V ,
so that


V
.
V0
In order to find the velocity of the piston we will use conservation of energy,
which states that the work done by the gas during the expansion is equal to the
kinetic energy gained by the piston. The work done by the gas is
"

V
 1 #
Z Vf
Z Vf
3
3
1
2 f
V 2
0
2
dV
=
P
V
W =
P 0 dV 0 = P V 2
=
2P
V
1

.
3
1
Vf
V 02
V 02 V
V
V
P0 = P

Problem OpaTh-F99-8 from Courses-SC.

The final volume is Vf = V + LS so that


"

W = 2P V 1

V
V + LS

1 #
2

and using W = M v 2 /2 we have v = (2W/M )1/2 and

v=

4P V
M

"


1

V
V + LS

 1 #! 21
2

(c) We will combine the ideal gas law with the adiabatic expansion law. Since the
ideal gas law gives P = nRT /V then we have that
P V = nRT V 1 = nRTf Vf1 ,
so that
Tf
=
T

V
Vf

1
=

V
=
Vf

so that
4V = V + LS
and
L=

3V
.
S

V
1
= ,
V + LS
2

Wedge of air

A wedge of air is formed between two glass plates held apart at one edge by a
sheet of paper whose thickness is 4.1 105 m. Green light (vacuum = 552 nm)
strikes the glass plates nearly perpendicularly. Assume nglass = 1.52, nair =
1.00.
(a) How many bright fringes occur between the place where the plates touch
and the edge of the sheet of the paper?
(b) Is there a dark fringe or a bright fringe where the plates touch? Why?
Solution
(a) The fringes occur because of interference between reflected rays 1 and 2 as illus-

trated in the diagram. Ray 1 is not inverted at the glass/air interface because
it is going from a more dense to a less dense medium. Transmitted waves are
always transmitted in phase. The reflected ray 2 is inverted at the air/glass
boundary, so the two reflected rays are out of phase by before we take into
account the path difference. We will get bright fringes when the path difference
is a half-integral number of wavelengths, so that
= (m +

1
)air , m = 0, 1, 2, ....
2

At the right edge the path difference is just twice the thickness t of the paper,
so there
1
2t = (m + )air ,
2
and we see that the last fringe has
m=

2t
1
2 4.1 105
1
=
= 148,
air
2
552 109
2

and since we start counting from zero we see that we have 149 bright fringes.
(b) As we saw above, if there is no path difference the two reflected rays have a phase
difference of and destructively interfere. This means that close to the contact
point we are in a dark fringe.
Problem OpaTh-F99-9 from Courses-SC.

Ideal gas in a cycle 2

In the cycle shown in the figure, 1 mol of an ideal gas ( = 1.4) is initially at
a pressure of 1 atm and a temperature of 0 C. The gas is heated at constant
volume to T2 = 150 C amd is then expanded adiabatically until its pressure
is again 1 atm. It is then compressed at constant pressure back to its original
state. Recall R = 0.082 L atm/(mol K). Find:

(a) the temperature T3 after the adiabatic expansion


(b) the heat entering or leaving the system during each process
(c) the efficiency of this cycle
(d) the efficiency of a Carnot cycle operating between the temperature extremes
of this cycle.
Solution
(a) To find the temperature T3 we will need the volume V2 = V1 and and the pressure
P2 before the expansion, which we can find using the ideal gas law. We have
V1 =
and so
P2 =

Latm
1 mol 0.082 molK
273 K
nRT1
=
= 22.4 L,
P1
1 atm

Latm
1 mol 0.082 molK
423 K
nrT2
=
= 1.55 atm.
V1
22.4 L

Now since the expansion is adiabatic to pressure P1 we can find V3 using the
rule that P V = constant, so that
V3 =
Problem OpaTh-F99-10 from Courses-SC.

P2
V ,
P3 1

and so

1/1.4
P2
V1 = (1.55)1/1.4 22.4 L = 30.6 L.
P3
Now we can find T3 using the ideal gas law,


V3 =

T3 =

P1 V3
1 atm 30.6 L
=
= 373 K.
Latm
nR
1 mol 0.082 molK

(b) Since there is no work done during 1 2 the heat entering the system is the
change in the internal energy of the system, which is
Q12 = CV (T2 T1 ).
We are not given the value of CV , but since = 1.4 = CP /CV = 1 + nR/CV
we know that CV = 5nR/2, and so
Q12 =

5
nR(150 K) = 30.75 L atm.
2

Since the expansion 2 3 is adiabatic, no heat enters or leaves the system. For
the compression 3 1 at constant pressure P1 we can use the definition of the
heat capacity at constant pressure
Q31 = CP (T3 T1 ) =

7
nR(100 K) = 28.7 L atm.
2

(c) The efficiency of the cycle is defined to be the net work done during the cycle
divided by the heat added during the phase when we add heat, which is 1 2,
so that
W
=
.
Q12
Note, however, that since the internal energy of the gas does not change after a
complete cycle, we must have that W = Q12 + Q31 , so that
=1+

28.7
Q31
=1
= 6.7%.
Q12
30.75

(d) An ideal Carnot cycle which operates between the temperature T2 = Thot and the
temperature T3 = Tcold (the maximum temperature during the heating phase
and the minimum temperature during the cooling phase, respectively) would
have an isothermal expansion at the hot temperature Thot and an isothermal
compression at the low temperature Tcold , with the other two processes being an adiabatic compression (to Thot ) and an adiabatic expansion (to Tcold ).
Isothermal expansions and compressions involve heats Qhot and Qcold which are
proportional to the temperature, through
 
Vf
Q = nRT ln
.
Vi
Once can show using the rule for adibatic expansions that the ratios of the
volumes at the beginning and end of the adiabatic processes are equal, so that the
ratio of the two heats is simply the ratio of the temperatures and the efficiency
of the Carnot cycle is simply
=1+

Qcold
Tcold
273
=1
=1
= 35.5%.
Qhot
Thot
423

10

Ideal gas in a cycle 3

The figure shows three reversible processes joining initial and final states of n
moles of a monatomic gas. For the three processes calculate:

(a) the work done on the system


(b) the heat interchanged with the environment
(c) the change in internal energy
(d) the change in entropy. Express all results in terms of n and the initial
temperature T0 .
Solution
(a) In what follows we will calculate the work done by the system (the usual convention) so that the answer to the question is always the negative of this.
Label the path from i to f which uses an expansion at constant pressure as path
1; it does work
Z
Z
W1 = P dV = P0 dV = P0 V0 = nRT0 .
Label the diagonal path as 2. To find the work done along this path we need
to know Pf , which we can find from the ideal gas law, since we know from the
third (curved) path that the final temperature has to be T0 . Hence
Pf =

P0 V0
P0
nRT0
=
=
,
2V0
2V0
2

Problem OpaTh-F99-11 from Courses-SC.

so that (using geometry to find the area)


W2 = V 0

P0 + P0 /2
3
3
= P0 V0 = nRT0 .
2
4
4

Finally, the third path is an isothermal expansion where P = nRT0 /V so that


Z 2V0
nRT0
W3 =
dV = nRT0 ln(2) = nRT0 ln(2).
V
V0
(b) The heat added to the system is the work W done by the system plus the change
in the internal energy of the system. For all paths the internal energy of the
system does not change since the initial and final temperatures are the same, so
in all cases the heat added to the system is simply the work done by the system.
(c) Zero in all cases since the initial and final temperatures are equal.
(d) The entropy changes by dS = dQrev /T when an amount of heat dQrev is added
at a temperature T in a reversible process. For an ideal gas we have
dS =

dU
dT
dT
dW
dV
dV
+
= CV
+P
= CV
+ nR
,
T
T
T
T
T
V

so that
Z

Tf

S = CV
Ti

dT
+ nR
T

Vf
Vi

dV
.
V

Since for all paths Ti = Tf we have that


Z 2V0
Z Vf
dV
dV
= nR
= nR ln(2).
S = nR
V
V
Vi
V0

Concepts 3
Provide a brief qualitative description for each item listed below.

(a) Compton effect.


(b) the difference between bosons and fermions.
Solution
(a) Compton scattering is the process of the scattering of a photon by an electron
(although it can also happen for other charged particles). The collision conserves
energy and momentum, so the electron recoils and the photon is emitted, in
general, at an angle to its original direction and with a reduced energy. The net
effect is that the photon appears to scatter from the electron with a reduced
final energy and so longer wavelength. The change in the wavelength depends
only on the scattering angle and not on the original wavelength.
(b) Bosons are particles like the photon which have integral spin (or intrinsic angular
momentum), while fermions are particles like the electron that have half-integral
spin. Fermions satisfy Fermi-Dirac statistics, because they obey the Pauli exclusion principle, which states that no two indistinguishable fermions can occupy
the same quantum mechanical state. Bosons satisfy Bose-Einstein statistics,
which means that they do not obey the Pauli exclusion principle.

Problem ModQM-F99-8 from 2003-Proficiency-Spring.

Concepts 4
Provide a brief qualitative description for each item listed below.

(a) Hyperfine structure in a hydrogen atom.


(b) Rayleigh scattering.
Solution
(a) The proton that makes up the hydrogen nucleus has a spin, and since it is a
charged particle it has a magnetic moment. The same is true of the electron.
These two magnetic moments interact with each other (one moves in the magnetic field generated by the other) and the resulting interaction Hamiltonian
cause a splitting of the ground state (and other energy levels) which depends on
the projection of the spin of the electron. The energy difference is very small,
of the order of 107 eV, and wavelength of the light from this transition is 21
cm, in the radio part of the spectrum, which is important for radio astronomy.
(b) Rayleigh scattering is a single-step process where a photon is absorbed by an
atom and promotes an electron into an excited state, which then decays back to
the ground state. The incident and scattered photons are correlated, and have
the same energy.

Problem ModQM-F99-9 from 2003-Proficiency-Spring.

Concepts exp obs

For the following pivotal experiments and observations in physics and astronomy, describe the key observation and why it changed our understanding of the
physical world:
(A) Blackbody radiation spectrum
(B) Photoelectric effect
(C) Compton scattering
(D) Rutherford scattering
(E) 2011 Nobel prize in physics to Saul Perlmutter, Brian Schmidt, and Adam
Riess
Solution
(A) The key observation was the peaking of the EM intensity at a temperaturedependent wavelength and its decrease at both shorter and longer wavelengths.
Max Plancks equation which reproduced the effect formed the foundation of
quantum mechanics by implying that light is only emitted in quanta of energy
hf .
(B) Key observation: The kinetic energy of each electron released from a metal surface
depends only on the wavelength of the light absorbed, not on its intensity. Albert
Einstein interpreted this to show that light is only absorbed in quanta of energy
hf . This provided a major confirmation of quantum mechanics.
(C) Key observation: The kinematics of the scattering of quanta of light off electrons
demonstrated the particle nature of light and confirmed the wave-particle duality
at the core of quantum mechanics.
(D) Ernest Rutherfords observation that alpha particles could scatter backward off
gold atoms negated the Thompson plum pudding model of the atom and led to
the understanding of the atom as a tiny, heavy nucleus surrounded by a cloud
of light electrons.
(E) Their careful measurement of the Hubble graph (distance vs. recession velocity)
of distant Type 1A supernovae revolutionized cosmology by showing that the
rate of expansion of the universe is increasing. This in turn implies that the
dominant energy of the universe is repulsive.

Photoelectric effect 2

Monochromatic blue light with a wavelength of 434.2 nm is incident on a


sample of cesium. Electrons emitted from the cesium surface are observed to
have velocities ranging up to 5.491 105 m/s. Note me = 0.511 MeV/c2 ,
qe = 1.602 1019 C, hc = 1240 eVnm.
(a) What is the work function for this sample of cesium?
(b) Explain why there is a range of emitted electron velocities.
(c) What is the wavelength of the fastest emitted electrons?
(d) Now assume that the hydrogen discharge lamp produces 2.0 W of power
radiated in this particular blue Balmer series spectral line. If the lamp
can be considered to be a point source and emits the light isotropically,
estimate how many of these blue photons per second strike a circular
cesium sample 7.5 cm in diameter and placed 10 cm from the lamp.
Solution
(a) The maximum kinetic energy of the emitted electron plus the work function of
the cesium is equal to the energy of the photon. This means that we can find
the work function by

=
=
=

hc
1
2
me vmax

2
1240 eV nm
1
5.491 105 2
0.511 106 eV (
)
434.2 nm
2
3.0 108
2.0 eV.
E T max = h Tmax =

(b) Electrons near the surface absorb all of the energy of the incident photon and
then leave the surface with the maximum energy available, which is the photon
energy minus the work function. Electrons deeper inside the metal are more
likely to interact with the electrons around other atoms on their way out of the
sample before leaving the surface, and these interactions will reduce their kinetic
energy.
(c) The de Broglie wavelength is
=

hc
1240 eV nm
h
=
=
= 1.33 nm.
p
mvc
0.511 106 eV(5.491 105 /3.0 108 )

(d) It is actually a difficult geometrical problem to find the area cut out of a sphere
by a circular disk which has its circumference touching the inside of the disk.
We need this to find the solid angle subtended by the cesium sample and so the
fraction of the emitted energy which is absorbed by the sample. If the radius of
the disk is small compared to that of the sphere we can approximate the solid
angle by the area of the disk divided by the surface area of the sphere which
goes through the center of the disk, but that is not the case here. This problem
Problem ModQM-F99-10 from 2003-Proficiency-Spring.

was not intended to be that hard, so we will use this approximation even though
it is not justified. The solid angle is, with this approximation,

2
r2
1 7.5/2
=
=
= 0.0351
4R2
4 10.0
and so the energy E per second which goes into this solid angle is this fraction
of the total emitted energy per second,
E = 0.0351Ptot 1 s = 0.03512106 J =

0.0702 106
= 4.381011 eV,
1.602 1019 J/eV

and the number of photons per second is this energy divided by the energy
h = hc/ per photon,
N=

2.19 101 1 eV
= 1.53 1011 photons.
1240 eV nm/434.2 nm

Compton scattering

A photon with wavelength 24.8 fm strikes a proton at rest. The photon


undergoes Compton scattering, and the scattered photon is seen by an observer
in the lab to be emitted at 180 with respect to the direction of the incident
photon. The mass of the proton is Mp = 938 MeV/c2 , and hc = 1240 MeVfm.
(a) What is the energy of the incident photon? What name would typically be
given to classify this type of photon? Give a very brief explanation for
your choice.
(b) Using relativistic kinematics, find (i) the wavelength of the scattered photon
and (ii) the de Broglie wavelength of the recoiling proton.
(c) If we could observe this reaction occuring in the center-of-mass frame instead of the lab frame, what would we then see as the difference between
the wavelengths of the incoming and scattered photons? Explain your
answer.
Solution
(a) The energy of the photon is
E = h =

1240 eV nm
hc
=
= 50 MeV,

24.8 10 6 nm

which is a gamma ray. Gamma rays are typically emitted during transitions
between excited states and ground states of nuclei, and typical nuclear excitation
energies are in the MeV range.
(b) We have to conserve relativistic energy and momentum in the collison. The initial
energy and momentum of the photon are related by E = pc, or we can write
that the photon four-momentum is (assume all motion is along the z-axis)
p = (E , p ) = (pc, 0, 0, p),
while the initial proton four momentum is
P = (mp c2 , 0, 0, 0).
The final photon four momentum is
p0 = (p0 c, 0, 0, p0 ),
since the photon back scatters, and by conservation of four-momentum (i.e.
energy and momentum) we know the final four momentum of the proton looks
like
P 0 = (mp c2 + pc p0 c, 0, 0, p + p0 ).
This takes care of conservation of energy and momentum, and we can now use
this to find p0 in terms of p if we use that the square of the proton four-momentum
is
P 02 = m2p c2
Problem ModQM-F99-11 from 2003-Proficiency-Spring.

which is equivalent to the relation


E 02 = P02 c2 + m2p c4 .
This gives the relation
=

(mp c2 + pc p0 c)2 (p + p0 )2 c2

m2p c4 + 2mp c3 (p p0 ) + (p p0 )2 c2 (p + p0 )2 c2

4pp0 c2

2mp c3 (p p0 )

p0

mp c(p p0 )
mp cp
mp c + 2p

p0 c

m2p c4

2pp

mp c2 pc
938
= 50 MeV
= 45.2 MeV
mp c2 + 2pc
938 + 100

so that the photon wavelength is




h
hc mp c2 + 2pc
938 + 100
hc
0 = 0 = 0 =
=
= 27.4 fm.
p
pc
pc
mp c2
938
Now we know that the momentum of the proton is p + p0 , so that the de Broglie
wavelength of the proton is
p =

hc
1240 eV nm
=
= 13.0 fm.
pc + p0 c
(50.0 + 45.2) MeV

General notes The energy momentum conservation for relativistic four vectors
is
P + p = P 0 + p0 .
(1)
Since the angel between initial and final photons is known and nothing is known
about the direction of the final proton it is convenient to rewrite this equation
as
P 0 = P + p p0 ,
(2)
and then take a relativistic square of both sides
m2p = m2p + 2P p 2P p0 + 2p p0 .

(3)

This leads to Comptons equation


0 =

m
mp + (1 cos )

(4)

for the frequency of the scattered photon.


(c) In the center of mass frame the initial momentum of the photon and proton must
be equal and opposite, and the same is true of the final momenta. Then the
conservation of energy condition looks like
q
q
pc + m2p c4 + p2 c2 = p0 c + m2p c4 + p02 c2 .
Since the left and right sides of this equation are the same function of p (or p0 )
then this can only be true if p = p0 , so the incident and scattered photons must
have the same momentum.

Decay of tritium 2
A vessel holds 2 g of tritium.

(a) What is the initial decay rate of the tritium?


(b) How much time will elapse before the amount of tritium falls to 1% of its
initial value?
Solution
(a) The number N of tritium nuclei as a function of time is
N = N0 e ln(2) t/t1/2 ,
so that the rate of decay is


ln(2)
dN
= N0
e ln(2) t/t1/2 ,
dt
t1/2
which at time t = 0 is
dN
(0) = N0
dt

ln(2)
t1/2


.

The total number of nuclei is the mass divided by the mass per nucleus, and the
half life is 12.3 y, so that


ln(2)
dN
2 109 kg
(0) =
= 7.13 108 s1 .
dt
5.01 1027 kg 12.3 365 24 3600 s
(b) If N/N0 = 1/100, then we have that
e ln(2) t/t1/2 =

1
,
100

or by taking the natural log of both sides


ln(2)
and so
t=

t
= ln(100),
t1/2

ln(100)
t1/2 = 81.7 y.
ln(2)

Problem ModQM-S00-2 from 2003-Proficiency-Spring.

Interference of coherent light

Four equally spaced coherent light sources with wavelength of 500 nm are
separated by a distance of d = 0.1 mm. The interference pattern is viewed on
a screen at a distance of 1.4 m. Finf the positions of the principal interference
maxima and compare their width with that for just two sources with the same
spacing.
Solution
Start by looking again at the case of two light sources. In this case when the path
difference d sin(), where is the angle from the normal to the plane of the light sources,
is an integral number of wavelengths we have constructive interference and so an
interference maximum. If this path difference is a half-integral number of wavelengths
we have destructive interference and so an interference minimum. This means that
the interference maxima are at angular positions
sin() = m

(maximum, two sources)


d

and the minima are at



sin() =

m+

1
2

(minimum, two sources),


d

so that the maxima have angular width equal to their angular spacing, /d.
In the case of four sources, the path difference between adjacent sources is still d sin(),
and the condition for a principal interference maximum (intensity 16 times that of a
single slit) is the same, but the condition for an interference minimum changes. There
is obviously an interference maximum for d sin() = 0; the first minimum will occur
when the four phasors from each source form a square (see diagram), and so the phase
angle between adjacent phasors is 90 . This means that we have
d sin() =

(first minimum, four sources).


4

By the same idea there are obviously interference minima when d sin() = /2 and
3/4. In between these angles we have secondary maxima (where the intensity is 1/16
times that of the principal maxima). The next principal maximum occurs when the
phase angle between each phasor is again zero, so d sin() = .

This means that the principal interference maxima are at angles

sin() = m , m = 1, 2, 3...
d
Problem S00Prof-11 from Courses-SC.

and have half the width that they have in the case of two sources. Converting this to
distances we can use the small angles approximation sin() = to write that positions
of the principal maxima
yn = L sin(n ) ' Ln = L

n 5 107 m
n
= (1.4 m)
= n(7 mm),
d
1 104 m

and so the principal maxima are separated by 7 mm, and their width is one half of
this distance, 3.5 mm. For two sources this width would be 7 mm.

Photoelectric effect 3

During a photoelectric effect experiment, sodium metal is illuminated with


light of wavelength 4.20102 nm. The stopping potential is found to be 0.65 V.
When the wavelength is changed to 3.10102 nm, the stopping potential is
found to be 1.69 V. Using only these data and the values of the speed of light,
c = 3.00 108 m/s, and the elementary charge, e = 1.60 1019 C, find a value
for Plancks constant.
Solution
Recall that in such experiments, photons are absorbed by the conduction electrons in
the surface of the metal, which then have enough kinetic energy that they are no longer
bound to the metal. The minimum energy required to remove an electron from the
surface (which is then moving with zero velocity and so no kinetic energy) is the work
function . This is measured by detecting a current flowing out of the surface through
a vacuum into an electrode, where a voltage is applied which drives the electrons back
into the metals surface. When this voltage reaches the largest kinetic energy the
electrons have been given from the photons the current stops.
From this and conservation of energy we know that
h = + eV,
where V is the stopping potential. If we have two frequencies and two values of the
stopping potential we can eliminate the work function by writing
h1 eV1 = h2 eV2 ,
so that
h(1 2 ) = hc(1/1 1/2 ) = e(V1 V2 ).
Solving for h we find
h

=
=
=

e(V1 V2 )
c(1/1 1/2 )
3.0

108

1.6 1019 C (0.65 1.69) V


m/s (1/4.2 107 1/3.10 107) m1

6.6 1034 J s.

Problem ModQM-F00 from Courses-SC.

Beta decay

In nuclear beta decay, electrons are observed to be ejected from the atomic
nucleus. Assume that electrons are somehow trapped within the nucleus and
that occasionally one escapes.
(a) Estimate the kinetic energy that such an electron must have. Assume a
nuclear diameter of 1.0 1014 m.
(b) Electrons emitted from the nucleus in nuclear beta decay typically have
kinetic energies of about 1 MeV. Comment on the difference between the
actual kinetic energy and your result for part a.
Solution
(a) The statement of this problem is confusing. In beta decay electrons are ejected
forcibly from the nucleus, since they actually come from beta decay of a neutron
into a proton and an electron (and an anti-neutrino), which releases energy. A
part of this energy is in the form of rest and kinetic energy of the electron,
another (smaller) part goes into the recoil energy of the proton and the neutrino
energy. Since for a free neutron about 1.3 MeV is available in total for this,
and we need 0.5 MeV for the electron rest energy, one might guess that in the
absence of all other effects the kinetic energy of the electron would be very
roughly one MeV. However, one also has to take into account the difference in
the nuclear binding energy of the initial and final nuclei, and nuclear structure
effects (the initial neutron may need to be in a higher energy level than the final
proton because of the Pauli principle and the fact that large nuclei have more
neutrons than protons). In addition, the ejected electron is attracted to the
positive charge of the nucleus and so a (possibly substantial) part of its initial
kinetic energy is required to overcome this Coulomb barrier if it is to become
free.
What we are asked to do is to suspend all of our knowledge of the physics of
this process and imagine an electron which is confined within the nucleus all
of the time. Since we are not given anything except the size of the nucleus, in
particular we do not know its charge, we cannot evaluate the energy required
to overcome the Coulomb barrier. If all we know is the size of the (roughly
spherical) nucleus which contains the electron, then all we can do is to estimate
its momentum and so kinetic energy using the uncertainty principle.
If the electron is confined to a region of size 1014 m, then by the uncertainty
principle it must have a x smaller than this, but how much smaller? This
is the essential problem with using the uncertainty principle for doing physics.
I got curious about this and decided to find x for a particle in its ground
state in a one-dimensional box x [a, a], which has wavefunction (x) =

(1/ a) cos(x/2a). The result is that


Z

2
( 2 6)
1 +a 2
x =
x cos2 (x/2a) = a2
,
a a
3 2
Problem ModQM-F00-3 from Courses-SC.

which since hxi = 0 gives


p
a
x = hx2 i =

2 6
= 0.361 a.
3 2

So if, as is usually done, we say that the uncertainty x in the electrons position
is the width 2a of the box, we make a mistake of a factor of 2a/(0.361a) = 5.5
in x, and so greatly underestimate p. Perhaps this approach to estimating
particle energies should be called the uncertain principle, good for an order of
magnitude ar best.
If we forge ahead regardless of this problem and use the diameter d of the nucleus
for x then we will very roughly estimate the momentum of the electron in the
nucleus by using
p

/2
h
197.32 (MeV/c) fm
h

hc

MeV
=
=
=
= 9.85
.
x
2d
2dc
20 fm
c

where we have used the convenient unit h


c = 197.32 MeVfm and that the
nuclear diameter given is 10 fm (1 fm=1015 m).
Since pc is larger (by a factor of 20) than the rest energy 0.511 MeV of the
electron we know that this is a relativistic electron, and we have to use the
relation (assume, again probably erroneously, that we can use p for the r.m.s.
momentum)
E = (p2 c2 + m2 c4 ) = 9.86 MeV ' pc.
(b) See all the physics reasons above for why this is bound to be a poor estimate of
the energy of a beta-decay electron.

10

Double-slit interference

Two slits of width a = 0.015 mm are separated by a distance d = 0.06 mm


and illuminated by light of wavelength = 650 nm. How many bright fringes
are seen in the central diffraction maximum?
Solution
The condition for a minimum of the diffraction pattern is
a
n
sin() =
,
2
2

n = 1, 2, 3...,

where a is the slit width. The condition for a maximum of the interference pattern is
d sin() = m,

m = 0, 1, 2, ...,

where d is the slit spacing. Since in this case d = 4a, we have that the m = 4
interference maximum occurs at the position of the n = 1 diffraction minimum. That
means that there are only three interference maxima on either side of the central
diffraction, not counting the central (m = 0) maximum, since the fourth maximum
will be wiped out by the diffraction pattern. This gives seven bright interference fringes
within the central diffraction maximum, regardless of the wavelength of the light.

Problem OpaTh-F99-2 from Courses-SC.

Definitions

Define the following properties of a solid, and explain how each can be measured
(a) electrical resistivity
(b) magnetic susceptibility
(c) specific heat
(d) thermal conductivity
(e) dielectric constant
Solution
(a) Electrical resistivity: , the constant of proportionality that relates R = (L/A),
where L is the length of a wire, A is the cross-sectional area, and R is the
resistance. To measure the resistivity of a conductor, you would measure the
length of the wire, the cross-sectional area, and then the resistance through
V = IR or R = V /I (V and I are the voltage and current, respectively).
(b) Magnetic susceptibility: = M/B, where M is the magnetization, and B is the
applied magnetic field. The magnetic susceptibility can be measured in several different ways - the Gouy balance is one method, where you can measure
the torque in a magnetic field of a material. The magnet remains stationary
while the sample moves, giving an apparent weight loss or gain. Another way
is through SQUID magnetometry (Superconducting Quantum Interference Device), which measures changes in the magnetic flux quantum.
(c) Specific heat: the amount of heat required to raise the temperature of either 1 g
or 1 mol of a substance by 1 K: c = (1/m) Q/T (m is the mass or molar
mass, Q is the amount of heat, T is the temperature). You can measure the
specific heat by adding heat to a known amount of substance and then measure
the temperature change. Of course, everything must be well insulated to prevent
excess heat loss or gain to the sample.
(d) Thermal conductivity: the constant of proportionality k that relates the heat flow
in a certain amount of time to the value of the heat gradient across an object of
cross-sectional area A.
Q
T
= kA
t
x

(1)

Here, Q is the amount of heat that flows in time t, the rod has cross-sectional
area A, and the gradient is T /x. To measure this, you would need to establish
a temperature gradient across a cylindrical object that you wish to measure (of
length x and cross-sectional area A), and then you would need to measure the
heat flow as a function of time. These experiments are done in thermally isolated
containers to prevent heat loss from other sources.
Problem 9 from Qualifying exam 2007-spring.

(e) Dielectric constant: this is the factor by which the internal electric field of a
material changes when an electrical field is applied: = s /0 where s is
the static permittivity and 0 is a constant. This can be measured through a
capacitor, where the capacitance is C = 0 A/d (A is the cross-sectional area
and d is the distance between the plates). In metals, the dielectric constant can
be infinite, and so other methods are needed.

Ideal gas in a cycle 5

Consider an ideal monatomic gas consisting of N particles each with mass m.


The gas is first expanded from volume V1 to volume V2 at constant pressure so
that P1 = P2 . The gas is then further expanded isothermally to a final volume
V3 with pressure P3 . The corresponding temperatures are T1 , T2 and T3 , with
T2 = T3 .
(a) Using classical thermodynamics compute the total entropy change and
express it in terms of V1 , V3 , T1 , T3 . (Recall that the heat capacity of
an ideal monatomic gas at constant pressure is CP = 52 N kB , where kB is
Boltzmanns constant).
(b) Using the partition function of the ideal classical monatomic gas, calculate
the Helmholtz free energy of the system as a function of particles N , the
volume V , and the temperature T .
(c) Using the Helmholtz free energy obtained in part (b), obtain an expression
for the entropy of the system. Use this expression to compute the total
entropy change of the gas and compare your result to that obtained in
part (a).
Solution
(a) For an ideal monatomic gas CV = 32 N kB and CP = 52 N kB . For the constant
pressure expansion the change of the entropy of the gas is
Z T2
CP
T2
T2
5
S12 =
dT = CP ln
= N kB ln
.
(2)
T
T1
2
T1
T1
For the isothermal expansion the internal energy of the gas does not change.
This means the heat added is equal to the work done by the gas.
Z V3
Z V3
V3
N kB T
dV = N kB T ln
.
(3)
Q23 =
P dV =
V
V2
V2
V2
Thus the change in entropy is
S23 =

Q23
V3
= N kB ln
.
T
V2

(4)

Using the fact that T2 = T3 , and V2 = V1 T2 /T1 we have for the total change in
entropy
3/2

S =

T V3
T3
V3 T1
5
N kB ln
+ N kB ln
= N kB ln 33/2 .
2
T1
V1 T3
T1 V1

(5)

(b) The partition function for an ideal monatomic gas is


QN =

1 N
Q1
N!

Problem 1 from Qualifying exam 2008-Fall.

(6)

where
Z
Q1 =

d3 rd3 p p2 /(2mkB T )
e
=V
h3

2mkB T
h2

3/2
.

(7)

The Helmholtz free energy is then


A = kB T ln QN = kB T N

N
ln
V

h2
2mkB T

3/2
1

(c) Given the Helmholtz free energy, the entropy is




A
S =
T V,N
!

3/2
5
V 2mkB T
+
.
= kB N ln
N
h2
2

(8)

(9)
(10)

It follows that the change in entropy is


3/2

S13 = S(N, V3 , T3 ) S(N, V1 , T1 ) = kB N ln


in agreement with Part (a).

V3 T3

3/2

V1 T1

(11)

Spaceship

A spaceship is sent from earth to another solar system which is at a distance of


20 light years from the earth. The spaceship is rapidly accelerated to a velocity
(relative to the earth) of 0.99c toward its objective. Upon reaching the distant
solar system, the crew takes some pictures as the ship is rapidly accelerated to
0.99c heading back toward the earth. The pictures are immediately transmitted
by radio to those waiting on earth. Using special relativity and ignoring the
relativistic effects of the periods of acceleration and deceleration:
(a) How long does the round trip take according to people waiting on earth?
(b) How long does the round trip take for people on the spaceship?
(c) After how many years since takeoff do the people on earth see the first
pictures from the distant solar system?
(d) In those pictures, they see the wonders of the distant solar system as well
as the clocks on the spaceship. What time do these clocks read (in years)?
Show that this is consistent with the Doppler shifted clock frequencies.
Solution
(a)
t = 40/0.99 = 40.4years

(12)

(b)

1
p
= 1/ 0.02 = 1/0.14
1 v 2 /c2

(13)

t0

t/ = 40.4 0.14 = 5.7years.

(14)

(c)
t = 20/0.99 + 20 = 40.2years
(d) Time=5.7/2 years2.8 years.
Doppler shift:
s
r
1 v/c
0.01
=
0 =
0 0 /14
1 + v/c
1.99

(15)

(16)

so time= 40.2years/14 2.8years, which is consistent with the above calculation.

Problem 8 from Qualifying exam 2008-Fall.

Estimations

(a) The molten metal in a furnace appears to emit predominantly blue light.
Is the temperature of the metal closest to (I) 3 K, (II) 3 104 K, or (III)
3 106 K?
(b) Estimate the ground state energy of a harmonic oscillator using the uncertainty principle.
(c) In an electron interferometer, an electron is split into two paths ABC and
ADC in the figure below and recombined. The rectangular paths have
the dimensions AB = DC =a and AD = BC =L. The whole rectangle is
placed in a uniform electric field along the direction AB or DC. If interference fringes are produced as the electric field strength is varied, find the
interference fringes separation as the change of the electric field in terms
of the electron kinetic energy, its mass and charge, the dimensions of the
paths and any universal constants.

Solution
(a) Visible light 2eV 2.4 104 K. Therefore, it must be case II.
(b) For the harmonic oscillator: (using px
h)
E=

(p)2
p2
1
1
h2

1
+ kx2
+ k(x)2
+ k(x)2
2m
2
2m
2
2
2m(x)2

In the ground state, find the minimum energy by setting dE/dx = 0:


 2 1/4
h
2
h

+ k (x) = 0 (x)4 = km
x = km
q
q


1/2

2
1
h
2
h
2 km 1/2
h
2
k
k
E = 2m(x)
+ 21 k km
= h2 m
+ 12 h
m
=
h
2 + 2 k(x) = 2m
h
2
dE
dx

2
h2
2m(x)3

Problem 6 from Qualifying exam 2008-Spring.

(c) Along BC:


2 2

h
k
K =q
+ eEa
2m
k = (K eEa) 2m
h
2

The phase is kL.


Along AD:
2

h
k0
K=q
2m
k0 = (K) 2m
h
2

The phase is k0 L. So the phase difference = (k k0)L:


!
r
r
2m
2m
(k k0 )L =
(K eEa) 2 K 2 L
h

Rewrite this as:


r
(kk0 )L =

2m
K 2
h

!
r
r
eEa
2m
eEa
2m eEa
eEaL
(1
) 1 L K 2 (1
1)L K 2 (
)L
k0
K
2K
2K
2K
h

What electric field is needed to shift one fringe? This is the amount needed to
change the phase by 2:
r
4 K
4
K
h2
eEaL
k0 = 2 E =
=
(k k0 )L
2K
eaL k0
eaL
2m

Thin fiber

A thin fiber of length L is stretched between two supports. The speed of


propagation of transverse waves on the fiber is c for both polarizations.
(a) What is the contribution of these modes to the heat capacity of the fiber
at low temperatures, assuming hc/L  kB T ? (Dont worry about solving
the integral to find the dimensionless prefactor)
(b) What is the heat capacity for hc/L  kB T ?
Solution
(a) Boson modes: k = n/L, (n = 1, 2, 3...). Let = kc, then
E=2

.
eh /kB T 1

(17)

For h
c/L  kB T ,
2L
E

Z
dk

eh /kT

2L
=
1 c

eh /kT


2 Z
kT
x
2L
h
dx x
=
h

e 1
1 c
0

Letting x =
h/kB T and solving for the heat capacity:
C=

E
4LkB 2

T
T

hc

C T as expected for 1D bosons.


(b) For
hc/L  kB T , we have:
E=2

X (
X n
hc/L)n
h

hc hc/LkT
e
=2
2
L
eh /kT 1
ehcn/LkT 1

At low temperatures, only the n = 1 mode is occupied, so we have:


E2

hc hc/LkT
e
L

And the heat capacity is:


C=

E
T

 2 hc/LkT

hc 2
e
L
kB T 2

Problem 4 from Qualifying exam 2009-Fall.

Estimations 2

For each of the following energy quantities, give an expression in terms of


fundamental constants which is dimensionally correct (numerical prefactors are
not required) and compute from your expression an order of magnitude estimate
in units of electron Volt (eV).
Useful information (formulas in Gaussian cgs units):
o Rydberg: 1 Ry = me e4 /(2h2 ) 13.6 eV.
o Bohr Magneton: B = e
h/(2me c) 6 105 eV/T(T Tesla).
o Fine Structure Constant: = e2 /(hc) 1/137.
Example: The electron kinetic energy in the ground state of the Hydrogen atom.
Solution: Denote the size of the atom by aB . From Newtons second law one
finds T = me v 2 /2 = e2 /(2aB ) for the kinetic energy and the quantization condition me vaB = h
leads to T = h
2 /(me a2B ). When equated with the potential
2
energy e /aB , this yields aB = h
2 /(me e2 ) 5 1011 m. The kinetic energy
2
4
is then T = me e /
h = 2Ry 10eV. (One Ry is 13.6 eV, but the order of
magnitude 10 eV is sufficient for this problem).
(1) The ground state binding energy of the electron in a U 91 ion (a Uranium
ion with atomic number 92, atomic weight 238, which is 91-times ionized
so that there is only one electron present).
(2) The ground state energy level splitting in a Hydrogen atom due to the
spin-orbit interaction (fine structure splitting).
(3) The hyperfine splitting in a Hydrogen atom (the splitting of electronic
energy levels due to the interaction with the magnetic moment of the
proton).
(4) The relativistic correction to the binding energy of a Hydrogen atom.
(5) The Zeeman splitting of electron levels in a Hydrogen atom when the
magnetic field is 1 T (T Tesla).
(6) The energy of a nucleon in a typical nucleus.
(7) The rotational energy of an H2 molecule.
Solution
(1) Using the scaling e2 Ze2 with Z = 92 in the expression for the hydrogen atom
we obtain
mZ 2 e4
104 Ry 105 eV.
E
h2

(2) This is the energy of the electron spin dipole in the magnetic field due to its
orbital motion (Er radial electric field, p momentum)
E B

vEr
e
h
pe
e8 me

2 Ry 103 eV.
c
me c me ca2B
4 c2
h

Problem 7 from Qualifying exam 2009-Spring.

(3) It is reduced by a factor of the mass ratio from the fine structure splitting
E
(4) By Taylor expansion of

me 2
Ry 106 eV
mp

p
m2 c4 + p2 c2

E me c2 (

p2 2
) 2 Ry 103 eV.
m2e c2

(5) The Zeeman energy in the magnetic field B is


E B B 104 eV.
(6) In a nucleus of radius rN 1fm = 1015 m(T kinetic energy)
EN TN =

TN
me r2N
h2

10eV =
10eV 107 eV.
2
mN rN
Te
mN a2B

(7) The rotational energy is (L angular momentum, reduced mass)


Erot

L2
me
h2

Ry 102 eV.
r2
mH a2B
mH

Photon gas

(a) Consider a photon gas at temperature T = 2.9 K. Using E = uV and


dE = P dV for an adiabatic, quasistatic process, derive an expression for
how the energy density u depends upon the volume V for an adiabatic,
quasi-static expansion of the gas. Hint: P = u/3 for a photon gas.
(b) For a photon gas, u T 4 . Using this, derive an expression for how the
volume V depends upon the temperature T for an adiabatic, quasistatic
process.
(c) If the universe expanded quasi-statically and adiabatically from an initial
state where the radiation temperature was T = 3000 K to the current
state with a temperature of T = 2.9 K, what is the ratio of the current
volume to the initial volume? Assume that the universe is composed of
no matter.
Solution
(a)
dE = P dV
E = uV dE = udV + V du = P dV = (u/3)dV
Therefore:
(4/3)udV = V du, or (du/u) = 4/3(dV/V)
This results in:
uV 4/3 = constant, or u V4/3
(b) Using what we have above, T 4 V 4/3 = constant, or V T3 .
(c) (Vf inal /Vinitial ) = (Tinitial /Tf inal )3 = (3000/2.9)3 = 1.1 109 .

Problem 8 from Qualifying exam 2009-Spring.

Estimations 3
Estimate each of the following, with an explanation for your reasoning:

(a) the average kinetic energy of a monatomic air molecule in this room.
(b) the molar heat capacity at constant volume of a rock at room temperature.
(c) the electrical resistance of a metal wire one meter long and 1 mm in radius.
(Hint: 108 m).
(d) the number of atoms in 1 cm2 on the surface of a solid metal.
(e) the number of atoms in 1 m3 of gas at room temperature and atmospheric
pressure.
Solution
(a) 3/2kT = 6.2 1021 J(for T 300 K)
(b) CV = 3R = 24.9J/molK
(c) R = (L/A) = 108 /( 106 ) = 0.3 102 3 103 Ohm
(d) Interatomic spacing of solids is 41010 m. Each atom then occupies an area on
a surface of roughly 1.9 1019 m2 . For 1 cm2 , which has an area of 1 104 m2 ,
we then have: number of atoms = 1 104 m2 /1.6 1019 m2 = 6.25 1018
atoms
(e) P V = N kT N = P V /kT = (101103 Pa)(1m3 )/(1.381023 300) = 2.41025

Problem 9 from Qualifying exam 2009-Spring.

Ideal gas in a cycle 6

One mole of an ideal monatomic gas at an initial volume V1 = 25L and pressure
P1 = 105 Pa is subjected to the following three step cycle.
First, it is heated at constant volume to P2 = 2 105 Pa.
Second, it is then isothermally expanded to V3 = 2V1 .
Third, the volume is reduced back to V1 at constant pressure.
All processes are quasi-static. The gas constant is 8.314J/(mole K).
(a) Draw the P V diagram of the cycle indicating the pressure and volume
after each step.
(b) Find the temperature of the gas at each step of the cycle.
(c) Find the heat flow for each part of the cycle.
(d) Calculate the efficiency of the cycle.
Solution

(a)
P1 = 105 Pa, V1 = 25L, P2 = 2 105 Pa, V2 = 25L, P3 = 105 Pa, V3 = 50L.
(b)
T1 =

P1 V1
105 Pa 25L
=
= 301K
J
R
8.314 molK
1mol
T2 = 2 T1 = 601K
T3 = T2 = 601K

(c)
Z
3
RnT, W = P dV
2
3
1 2 : V = const, W = 0, Q12 = RT = 3750J
2
Z
V3
2 3 : T = const, U = 0, W = PdV = nRT ln
= 3466J
V2
Q23 = W = 3466J
Q = W + U, U = CV T, CV =

Problem 1 from Qualifying exam 2010-Fall.

3 1 : P = const, W = PV = 2500J
3
U = Rn 301K = 3750J
2
Q31 = W + U = 6250J
(d)
efficiency =

Qh |Qc |
W
Q31
=
=1
= 0.134
Qh
Qh
Q12 + Q23

10

Thermal neutron

The nucleus 113 Cd captures a thermal neutron having negligible kinetic energy,
producing 114 Cd in an excited state. The excited state of 114 Cd decays to the
ground state by emitting a photon. Find the energy of the photon. Useful
constants:
me = 0.511 MeV/c2

mn = 1.008665 u

m(113 Cd) = 112.904401 u

m(114 Cd) = 113.903359 u

1 u = 931.5 MeV/c2

1 eV = 1.6 1019 J

Solution
The reaction is 113 Cd + n 114 Cd + .
4-vector conservation with c = 1


p 2
m113
mn
m114 + p2
0 0
p



0 + 0 =
0
0
0
0
m113 + mn =

p
p
+

0
0

m2114 + p2 + p

(recovering c)
E

=
=

Problem 3 from Fall02.

2m113 mn m2114 + m2113 + m2n 2


c
2 (mn + m113 )
9.042M ev.
p =

You might also like